Этого треда уже нет.
Это копия, сохраненная 29 сентября 2019 года.

Скачать тред: только с превью, с превью и прикрепленными файлами.
Второй вариант может долго скачиваться. Файлы будут только в живых или недавно утонувших тредах. Подробнее

Если вам полезен архив М.Двача, пожертвуйте на оплату сервера.
Тред тупых вопросов №108 VAN ALLEN EDITION 489400 В конец треда | Веб
Тред вопросов о жизни, Вселенной и всём таком.

Спрашиваем то, за что в других местах выдают путёвку в биореактор. Здесь анонимные учёные мирового уровня критически рассмотрят любые гениальные идеи и нарисованные в Paint схемы.

Предыдущий тут: >>484908 (OP)
https://2ch.hk/spc/res/484908.html (М)

Q: Можно быстрее?
A: Можно упасть в пузырь альбукерке, наса уже почти надула его.

Q: Я начитался охуительных историй про уфологию, че делать, нам жопа?
A: Да, тебе жопа, можешь сгонять в зогач или куда оттуда пошлют.

Q: Что будет с человеком в вакууме без скафандра / если он упадет на черную дыру / попробует ступить на поверхность газового гиганта/солнца?
A: Он умрёт.

Q: Почему бы не привязать ракету к воздушному шару или стартовать с горы?
A: Космос - это не как высоко, а как быстро, большая часть энергии ракеты уходит на разгон вбок.
Подробнее тут https://what-if.xkcd.com/58/ (английский) https://chtoes.li/orbital-speed/ (перевод)
2 489406
Ебанный стыд...
Во-первых, Алькубьерре.
Во-вторых, не упасть, а создавать вокруг корабля изнутри (иначе кина не будет).
В-третьих, НАСА искривляет пространство на десятимиллионную часть, контролируя это сверхточными интерферометрами, до самого варп-привода здесь - как до Антарктиды раком.
3 489407
>>89400 (OP)
Аноны, подскажите, что за игра космосим, инди с процедурными планетами, управляешь косм кораблем, можешь из него выйти на планету.
Это не rodina, похожа на space enigine. И не no man sky
4 489408
Можно ли на основе Старлинков наклепать много-много дешевых однотипных АМС, вывести их старшипом, а затем они посредством анальных пертурбативных маневров и собственных ионников разлетятся по своим целям (скажем, околоземные астероиды, астероиды основного пояса, короткопериодические кометы)?
5 489413
>>89408
Зачем?
6 489416
>>89413
Чтоб все ахуели, очевидно же. Или Старлинки недостаточно радиационностойкие для этого вашего глубокого касмаса?
7 489419
С какой скоростью надо лететь, чтобы реликтовое излучение стало видимым из-за синего смещения?
8 489420
>>89408
манёвры это масса
вменяемые датчики/сонсоры/камеры это дораха, долга и мала
старлинк это джва центнера, немного мелочи и вечерок наколенке
алсо в массовые ширпотребные амс так никто и не вышел(
9 489424
>>89419
Чтоб увеличить энергию фотонов до видимого света, надо поднять их энергию раз этак в 1000 (цветовая температура увеличится с 2,7 кельвинов до уже видимых 2700).

Лоренц-фактор в 1000 это примерно 0.9999995 c, дохуя быстро выходит.
10 489427
>>89424
Ладно, пока отложим, значит.
11 489430
>>89424
А насколько оно однородно? Если разогнаться в манямечтах до этой скорости, впереди я буду видеть однородный фон, или какие-то полутона можно будет различить?
12 489433
>>89430
Супер однородно, при температуре 2700К размах неоднородностей не дотянет даже до одного градуса.

Кстати, однородного фона же тоже не будет, эффект же будет максимален только в точке прямо по направлению движения, а в стороны от нее видимая «температура» будет падать, переходя во все более красные и тусклые цвета вплоть до полной черноты. Будет этакий радиальный градиент.
13 489437
>>89400 (OP)

>Pickering, van [...].jpg


Это мокет?
14 489439
>>89437
да, но в натуральную величину
15 489445
Можно ли светить в свой солнечный парус лазером с этого же корабля и разгоняться, как волк из "ну погоди" на лодке?
16 489446
>>89445
Да, если твой парус отражает фотоны.
17 489458
>>89445
можешь не изъебываться и просто светить в другую сторону
18 489466
>>89430
если разгонишься - градиент анизотропии будет куда заметнее, но поскольку это излучение чт - это всё равно будет просто светлое пятно, темнеющее и краснеющее до границ передней полусферы
1480174367310-0.jpg89 Кб, 604x562
19 489468
>>89424
1. Я правильно понимаю, что обычный видимый свет от свёзд при такой скорости превратится в сверхжёсткую гамму, выжигающую всё к хуям собачьим, и это накладывает некий фундаментальный предел на околосветовые путешествия?

2. Более сложный вопрос. Известно, что не может существовать фотон, чья длина волны меньше планковской длины - иначе он сразу же превращается в чёрную дыру. Но представим, что нам навстречу летит очень борзый фотон, а мы летим с околосветовой скоростью, и приведённая длина волны этого фотона получается меньше планковской длины. Что это будет за аномалия, увидим ли мы сингулярность, и может ли такой сверхпланковский фотон распидорасить пространство-время при прямом попадании в околосветовое тело?
20 489470
>>89468

> 2


преобразование лоренца
21 489474
>>89458
Волк тоже мог просто в другую сторону дуть?
22 489493
>>89407
Может быть Elite Dangerous? Я больше не знаю никаких космосимов с высадкой на процедурные планеты.
23 489494
>>89458
Но ведь скорость света постоянна, и не зависит от скорости движения корабля. Разве такой фонарик в парус не был бы куда эфективнее дующего волка?
24 489498
>>89406

>Во-первых, Алькубьерре.


Фига се, ты наверное всю азбуку Даля прочитал. ICQ не жмет?
>>89494
При чем тут скорость света? Энергия фотона у нас скоростью его выражается теперь? Вон уже тонко намекнул один, что можно светить в парус, если он будет отражать назад, или светить сразу назад, как указал другой, пожалев шестерни в голове анона.
25 489499
>>89407
Space engineers или empyrion или Planet Nomads, их много развелось
26 489502
>>89498

>Фига се, ты наверное всю азбуку Даля прочитал. ICQ не жмет?


Тебе самому не жмет, разговаривать с пастой, которая была в этих тредах уже несколько лет назад?
27 489503
>>89502
Не жмет. Захожу раз в полтора года, позоонаблюдаю деградацию тематик и ухожу, проблемс?
28 489515
>>89498

>Даля


Неуч, плез. Все же знают, что Брайля.
29 489527
>>89515
Посмотрите на него, я хуею, Луи Брайль куриц научился скрещивать разных мясных пород, это ты его с Бернулли путаешь.
30 489536
>>89527
Бернулли отказались к вам в дикую Россиюшку ехать, возьмите Эрдёша.
31 489621
>>89468

>превратится в сверхжёсткую гамму, выжигающую всё к хуям собачьим


А во что тогда превратится обычная гамма?
32 489625
>>89621
В еще более суровую гамму.
33 489659
Не совсем про космос. Какова должна быть планета и ее атмосфера, чтобы на ней было выгоднее летать на дирижаблях, чем на самолетах? То есть, чтобы аборигенам было известно и то, и другое, но самолеты по каким-то причинам были неэффективны. Плотная атмосфера, вроде как увеличит как аэростатическую (для дирижаблей), так и аэродинамическую (для самолетов) подъемную силу, так что этого недостаточно...
34 489664
>>89659
Из сайфача протек? Только честно!
35 489665
>>89664
Нет. Честно.
36 489668
>>89659
Думаю состав газов. Должна быть более тяжелая основа чем наш азот, чтобы вытеснение водорода/гелия было сильнее.
37 489685
если бы на марсе была азотно-кислородная атмосфера, идентичная в процентном соотношении земной, но при этом такой же разряжённой, там бы можно было дышать? скажем днём лета, когда марс ближе всего к солнцу и при условии, что мы не считаем считаем, что солнечной радиации, пыли и потенциальных вирусов тупа нет
38 489688
>>89685
Ты про тамошнее давление атмосферы слышал?
39 489690
>>89685
Дышать нельзя уже на Эвересте, точнее можно, но недолго. А там что-то около 30% нормального давления.
40 489692
>>89668
А если еще сила тяжести высокая? Аэростату хорошо (точнее, примерно похуй), а самолету хуево, так?
41 489696
>>89692
Думаю сила тяжести оказывает одинаковое влияния на эти типы аппаратов
42 489701
>>89621

>А во что тогда превратится обычная гамма?


Ну вот гамма и превратится в НЁХ с длиной волны меньше планковской. И нихуя не понятно, как такая штука будет взаимодействовать с веществом, если не дай бог в него попадёт.
43 489705
>>89701
Без подъебов, ты посчитал или так думаешь? Я попытался посчитать эффект Доплера для гаммы с частотой 10^24, скорости на 1 м/с меньше С, у меня получилось 10^28 всего порядок новой частоты (^29 для отличия на 0,001м/с, да даже для ультравысоких энергий с частотой ^28, будет ^33), насколько я обосрался? Длину получим поделив С на частоту, так? Т.е. ^-26 по максам. Это ведь все еще далеко от планковских 10^-35.
Наверное обосрался, если что, помогите посчитать правильно, с меня как всегда.
44 489725
Если на Марсе такая тощая рахитная атмосфера, то почему там бывают пылевые бури всеплатнетных масштабов?
45 489726
>>89725

>Если у меня тонкие пальцы, почему волосатые ноги-то?


Ты скорость ветра только что привязал к давлению, я правильно понял?
15605120578670.webm10,8 Мб, webm,
960x720, 0:50
46 489727
>>89726
Сам не знаю. Но если там атмосфера никакущая, то даже облаков там быть не должно - это же почти вакуум. Откуда там пылевые смерчи, облака и бури?
47 489728
>>89727

>это же почти вакуум


Пиздишь. Вот в космосе - почти вакуум, а там атмосфера. Было бы там 3 атома на км2, тогда да, а там и ветер, и бури. Не меряй по Земляшке все, станет сразу легче.
48 489729
>>89728
км3, конечно
починил
49 489739
>>89685
По земным меркам там практически вакуум
50 489743
Как добывать минералы в космосе? Там же полно ценных металлов необходимых во всех сферах производства и прочем. Не ужели профит с редких элементов не превысит все затраты? Ну и в конце концов с некоторыми элементами на Зеляшке вроде как проблемы, а они все равно нужны.
51 489745
>>89743

>Как добывать минералы в космосе?


Бурить, химией растворять, если надо - плавить. Технологически в невесомости свои заморочки, но ничего принципиально нового

>Не ужели профит с редких элементов не превысит все затраты?


Нет. По крайней мере золоту или платину невыгодно на землю отправлять, может совсем дорогие/редкие, если будет очень высокое содержание.

>Ну и в конце концов с некоторыми элементами на Зеляшке вроде как проблемы


Пока все проблемы решаемы прям тут, как видишь.
Есть смысл добывать, чтобы прямо там строить, возить, при нынешних ценах - не выгодно. Или перевозка должна стать гораздо дешевле (взлет, поиск, [буксир,] обработка, доставка и обслуживание всего этого),
52 489747
>>89745
Еще плюс ко всем проблемам собственно добычи, разведку-то как проводить? Просто отправить экспедицию бородатых дядек с рюкзаком тушенки не получится.
53 489748
>>89747
Двачую.
Была интерактивная карта с астероидами и содержанием, не помню где, потерял пару лет назад. Да, это тоже деньги и время, сами аппараты собрать дохуяллиард денег пока, с людьми если - это вообще пиздец сразу, еда, вода, воздух, безопасности, экраны, долгие перелеты, но не слишком, топляк...
Короче пока список почему "нет" получается в разы длиннее. Разве что поставить флаг, написать, "моя твоя астероид руда шатал" и улететь.
54 489749
55 489775
Если в нашей галактике в центре огромная черная дыра которая засасывает всё что жвижется тог каким образом ншегалактика расширяется со скоростью света? если дыра засасывает то как она расширяется?
56 489776
>>89775
дыра ничего не засасывает
57 489777
>>89775

>Если у меня в ванной есть слив, который иногда засасывает воду, а иногда не засасывает, какого хуя квартиру, воздух, небо и Аллаха туда не засосало? И почему по улице автобус ездит в направлении от слива, должен был только по направлению и недолго же?


Ну даже хуй знает.
58 489779
>>89777
ебать я проиграл.
Алсо вы вкурсе что у нас во вселеной уже несколько триллионов галактик на хуй? вы представляете каокй пиздец блять?
алсо есть ещё теория что вселенная не одна а вообще мультивселеная, я ещё крышей не поехал когда всё это представил. пиздец.
а вообще у меня моя теории в голове о том что посде смерти мы улетаем в дрпугую галактику за пол вселеной, типо потом там живём как духи элементы .
Я МЫ ВСЕЛЕННАЯ
59 489780
>>89779
Скорее грусть, потому как мы не очень хотим и свою систему обжить, а водородец-то горит, часики тикают, метеориты летают, динозавры костлявыми пальцами грозят, но нам похуй, вон у кого львы и пумы проснулись, у кого блэкливсмэттерс, а у кого и тупо средняя зп такая, что даже на водку еле хватает.
60 489805
>>89775

>в нашей галактике в центре огромная черная дыра


Эта черная дыра такая огромная, что весит аж меньше тысячной доли процента всей галактики.

>которая засасывает всё что жвижется


Она ничего не засасывает. Ну да, то что влетает в дыру - уже не вылетает. По орбите нарезать круги можно до посинения.

>тог каким образом ншегалактика расширяется со скоростью света


Галактика не расширяется. Тем более со скоростью света.

Еще вопросы?
61 489808
>>89805

>Галактика не расширяется.


чё? объекты друг от друга отдаляются со скоростью света после взрыва. разве нет?
62 489810
>>89808

>объекты друг от друга отдаляются со скоростью света после взрыва. разве нет?


Нет. Я как сидел в кресле, так и сижу.

Космологическое красное смещение - это пространство растягивается. На масштабах галактик это не заметить.
63 489811
>>89810
хуи сосёшь? бочку делаешь?
64 489813
>>89811
Делаю, сосу.
65 489816
>>89808
Вселенная и Галактика это немного разные вещи. Объекты друг от друга отдаляются тем быстрее, чем дальше они уже. В частности, близкие объекты друг от друга вообще не удаляются, потому что то крошечное удаление, которое есть (типа микрона за тысячелетие) легко компенсируется гравитацией.

Грубая аналогия. Возьми резиновую ленту и поставь на ней три точки: две рядом на одном конце и еще одну на другом конце. Теперь растяни ленту на 5 см за одну секунду. Дальняя точка уехала от других почти на 5 см, то есть она "удаляется со скоростью почти 5 см/с". А расстояние между двумя ближними точками изменилось ну скажем на 0,5 мм (зависит от того, насколько они близкие). То есть скорость удаления около 0,05 см/с, в сто раз меньше.
66 489825
>>89816
Если где-то образуется пузырек истинного вакуума, он никогда не достигнет другой галактики из-за этого общего расширения пространства?
67 489827
>>89825
Ты тут тупостью траллишь? Расширяется само пространство, везде и очень мало, для всех этих галактик и более крупных структур, удерживаемых вместе гравитацией, это как толстеющий анон в джинсах, джинсы на изи удерживают его жирноту, все лишнее вываливается только там, где джины кончились. Причем толстеет он так неспешно, что заметить это за 5 минут нельзя, но можно замерить за год (много-много световых лет расстояния, далекие галактики и сверхъяркие объекты). И однажды, видимо, настанет момент, когда эту толстоту уже не удержат джины и жопа порвет их нахуй (разлетятся структуры удерживаемые гравитацией), вываливаясь во все стороны (большой разрыв), но это пока только экстраполяция наших измерений, и будет это очень не скоро.
Так же, в связи с этим всем связано понятие видимой вселенной и объяснение почему этот размер такой. Это как жопа анона в кресле, на фото мы видим, что когда-то она в него влезала, теперь перестала (мы видим свет объектов, до которых уже не долетим, без заигрываний с перемещениями быстрее света, всеми этими варпами-хуярпами). Ведь скорость света конечна, т.е. можно найти такое расстояние, где этой жирноты (пространства) будет прибывать так много, что свет, вылетев сейчас, уже никогда не долетит до объектов на этом расстоянии через все эти миллиарды лет полета (млрды св. лет расстояния соответственно). Вот тебе сверхсветовое расширение, но это расстояние в пиздец раз больше, чем одна галактика.
68 489828
>>89827
Отныне я неймфаг.
мимо Стивен Жопинг
69 489844
>>89827
тебя тот анон про другое спрашивал, довен
70 489845
>>89844
Мелкобуква говорит внятно что хочет - человек делает остальное.
71 489846
>>89845
мамку ебал
72 489848
>>89845
слышь ты, великобуква, ты даже вопроса понять не можешь, хули с тобой разговаривать - у тебя iq отрицательный
74 489852
>>89849
сосать!
75 489853
>>89852
Соси, козлик, соси.
76 489858
>>89853
буду тебе, олигофрену, ебучему, который не знает про ложный вакуум, писать так мелко, что без залупы не прочитаешь
изображение.png499 Кб, 600x800
77 489862
>>89853

>Соси


>>89858

>буду тебе

78 490026
>>89862
Это тот самый дед ИВЦ?
7817640.jpg144 Кб, 496x586
79 490029
>>90026

>тот самый дед


Всегда проигрываю.
80 490069
Теория относительности, насколько я помню, вытекает из предположения (подтверждающегося наблюдениями), что существует некоторая конечная максимальная скорость c, быстрее которой ничто двигаться не может.

А если предположить что в какой-то другой гипотетической вселенной предельной скорости нет? Можно ли построить непротиворечивую физику в таком случае? Сильно ли она будет отличаться от нашей, и в чем?
81 490071
>>90069
Так это ж классическая механика. Никаких внутренних противоречий в ней нет, отказались от нее только потому, что что наша вселенная по-другому устроена.
82 490072
>>90069

>максимальная скорость c, быстрее которой ничто двигаться не может.


А как я помню, загвоздка была в том, что свет двигался с одной скоростью, независимо от движений источника/приемника, что было трудно объяснить зефиром, а еще, всякие Лоренцы уже ввели локальное время в уравнения, так что Альберт Конь просто обобщил все вот это. Ну и ученые быстро расписали, как переходить из одной системы координат - в другую. Про "не быстрее" это уже вывод, следующий из релятивистики, т.е. сначала относительность, а потом вот эти все построения.

>гипотетической вселенной предельной скорости нет?


Так 90% времени он и не нужен, этот предел. Пока мы ничего до С даже близко не разгоняем. Или ты хочешь, чтобы у тебя там свет мгновенно летал на любое расстояние?
83 490073
>>90071
Ну там не будет никаких проблем с законами сохранения, например? Когда свет излучается в соседней галактике и моментально поглощается тут, скажем. Ньютоны всяко о таком не задумывались же.
84 490076
>>90072

>Или ты хочешь, чтобы у тебя там свет мгновенно летал на любое расстояние?


Ну типа того (я собственно ничего не хочу, мне интересно). Просто если брать обычные галилеевские правила сложения скоростей (а хули, если максимальной скорости нет), то любая скорость должна быть достижима, просто надо взять быстрый поезд, а с него уже светить фонариком. То есть, если мы хотим сохранить некую "универсальность" света, то надо сразу ему бесконечную скорость давать.
85 490077
>>90073
Никаких, классическая механика внутренне непротиворечива.

Возможно, свет вообще не будет излучаться, поскольку окажется, что стабильных звезд при таких физических законах не образуется, но никаких парадоксов не возникнет.
86 490079
>>90077
Ну ладно. Там же, собственно, было два вопроса, и второй - "Сильно ли она будет отличаться от нашей, и в чем?". Если звезд не будет, то это довольно сильное отличие.
87 490080

>At this altitude the airplane will drop the external fuel tank and continue to climb near vertical to the edge of space at 61 mi (100 km). Excelsior will reentry into atmosphere and land as a glider.


http://www.arcaspace.com/en/iar111.htm
Это вообще возможно? Чтобы достичь такой высоты Space Ship two стартует с носителя.
Ну и румыны-эта ракетная Чёрная вдова с кабиной от Space Ship'а ещё и с воды взлетает!
88 490083
>>90080
Чот не увидел, что она там возить должна была. Space Ship two - 8 человек (2 пилота), а эта йоба только двух пилотов? Если она только себя на суборбитальную выводит, без полезной нагрузки - то почему бы и нет? Скорее тут технически сложно сделать так, чтобы было пофиг на высоту/давление, стартуй как самолет, а потом порхай как бабочка ракета.
В том же KSP, например, у меня херово всегда выходили космосамолеты именно поэтому, топляка запас маловат, для каждого режима, много лишнего в космос тащишь, все эти крылья, разве что реентри управляемый выходит и сажать относительно удобно.
89 490090
>>90080
Аркаспейс это фингербокс, которому много лет.
90 490099
>>90080
бред ебаный, форма просто подражание Ф-22/35 чтобы крута было, для суборбитального самолета не подходит вообще
91 490100
>>90083

>Чот не увидел, что она там возить должна была.



Он должен запускать вот эту ракету с высоты 17 км.
https://pt.wikipedia.org/wiki/Haas_2
https://www.youtube.com/watch?v=qRG7OlS1nCg
НО!

>The aircraft was designed to have a crew of two, the pilot and the navigator, or passenger, although ARCA was considering increasing the crew capacity up to five for space tourism



Кстати, если бы были шасси, а не поплавки, то ЛТХ улучшились бы?
92 490101
>>90099
yf-23 тогда уж.
93 490105
>>90080
А нахуя там вообще люди?
Серьезно. Дешевле было бы заебенить дрон.
94 490108
wqeqwewqeqwe
95 490133
Как так вышло, что при возрасте Вселенной - 13.799±0.021 миллиарда лет, этой звезде HD 140283 - 14.46 ± 0.8 миллиарда лет?

Она старше Вселенной?
96 490135
>>90133
Возраст звезды установили в 2013. Возраст Вселенной уточнили в 2015. Инструменты использовались разные.
97 490136
>>90133

>Due to the uncertainty in the value, this age for the star may or may not conflict with the calculated age of the Universe


Одно из измерений недостаточно точное.
98 490137
>>90133
14,46 - 0,8 = 13,66. Загадка века решена.

При этом ±0,8 это всего лишь одно стандартное отклонение, что означает, что есть 16% вероятность того, что возраст звезды даже меньше 13,66 млрд лет.

Если бы на 14,46 ± 0,08 замерили, тогда бы было сложнее объяснить.
99 490152
>>90137

>16% вероятность того, что возраст звезды даже меньше 13,66 млрд лет


Или больше 15,26.
3-howcanspacet.jpg83 Кб, 599x600
100 490153
>>90152
Для обывателя, или даже мамкиного космонавта из спейсача, совершенно нет разницы, 10, 13 или 16 млрд лет прошло с начала времен. Хорошая часть земного шара до сих пор верит, что мир создал б-г, или там в плоскую Землю и заговор жидорептилойдов.
101 490154
Если галактики в среднем удаляются друг от друга, значит ли это, что на значительном удалении от нас плотность галакик возрастает, и чем дальше тем больше?
102 490155
>>90154
Схуяли бы? Если ты разбавил водой суп в кастрюле, означает ли это, что в среднем суп стал гуще, чем дальше от ручек - тем гуще?
103 490156
>>90155
Не понял твою аналогию. Когда ты наблюдаешь удаленные (на миллиарды св. лет) галактики, ты их видишь не только далеко, но и давно. И, насколько я понимаю, тогда (среднее) расстояние между ними было заметно меньше, чем сейчас.
104 490159
>>90156
С этого и надо было начинать. Да, в прошлом вселенная была меньше, плотнее и горячее. На значительном расстоянии, если мы туда прыгнем прямо сейчас, должно быть примерно так же, неоднородно, но неоднородно примерно "как здесь".
105 490161
>>90159
Ну да, я имел в виду наблюдаемое состояние, а не то "как на самом деле".

А есть разница между "радиальной" и поперечной наблюдаемой плотностью? Насколько я понимаю, увеличение плотности должно наблюдаться в направлении луча зрения, но не поперек. Или нет? Сложно представлять полет луча в расширяющемся пространстве, мозги плавятся.
106 490162
>>90161
Представляй 2д на примере воздушного шарика. Между любыми точками, которые ты поставишь на нем маркером, расстояние будет расти примерно одинаково (пипку только не трогай, у вселенной ее нет).
107 490163
>>89705
Не сильно обосрался. Цифры не проверял, но на 1 м/с меньше с это еще далеко до реально охуевших релятивистских скоростей.

Посоны, ответьте про ЧД из фотонов на релявоскоростоях.
108 490165
>>89747
Наспамить дешевыми розеттами, очевидно же.
И науку нагенерят, и разведку проведут.
109 490166
>>90165

>дешевыми розеттами


Пока космос дешевый разве что по сравнению со стоимостью километра МКАД. А так-то да, массово начнем летать - будет проще.
110 490232
Что поменялось за год моего отсутствия тут?
111 490233
>>90232
Колонизировали Марс, строится корабль для полета на Вегу.
112 490236
>>90232
пораши больше стало, моча почти не трет
113 490237
>>90236
Это печально.
114 490265
Сможет ли человек без скафандра перебраться из одного шлюза в другой с помощью веревки? При этом расстояние между ними 10 метров, человек одет в обычную рубашку и штаны, предварительно плотно заткнет нозди, уши, зажмурится и выдохнет? Или шансов нет из-за ограниченной скорости декомпрессии?
115 490277
Как будет выглядеть нейтронная звезда с планеты в системе этой звезды?
А если система двойная и вторая звезда типа Солнца, что будет видно на небе?
116 490281
>>90277
Так же как и обычное солнце, только очень точечное за счет размеров. Но вообще обсервера быстро лучирует злыми лучами смерти нахуй.
117 490282
>>90265
Никаких шансов, потеряет сознание еще при декомпрессии в выходном шлюзе, как было с тем техником насовским. 14 секунд там что ли он был в сознании, и это не при плавной декомпрессии, а при резком переходе через лимит армрестлинга. https://en.wikipedia.org/wiki/Armstrong_limit
118 490313
Нахуя пытаются связаться с пришельцами? Вдруг сигнал перехватят какие-нибудь космические уебаны (Тираниды/роботы/фанатики), которым не в падлу будет прилететь на Землю всё сожрать?
119 490314
>>90313
Было бы охуенно. Мысль, что мы одни, как минимум в пределах взаимодействий со скоростью меньше С и сроками не более тысячи лет - пугает больше. Алсо будь мы космическими уебанами - нужно много условий, чтобы планета с высокоразвитой жизнью вообще была нам интересна в плане колонизации. Ну чтобы не случилось, как Уэльсовскими треногами, чтобы было вообще выгодно снаряжать военную экспедицию в другую систему и т.д.
120 490323
>>90282
14 секунд не хватит перебраться?
121 490326
>>90323
Не он. Самому вряд ли, но протащить пока мацой давится должно хватить времени. На той же веревке, теоретически. Ну если выбор только сдохнет гарантированно или вот так извращаться.
122 490328
Как (какими способами) вообще можно собирать межзвездный водород, если корабль летит на релятивистских скоростях?
Как оценить до какой скорости будет эффективен термоядерный двигатель?
Что там со скоростью истечения, будет ли излучение вносить заметный вклад в тягу?
И если межзвездный водород хорош для самой реакции, можно ведь домешать туда что-то потяжелее, для увеличения импульса, вот это потяжелее только с собой везти?
123 490330
>>89827

> больший разныв


> экстаполяция наших измерений


че у вас там за ваши измерения такие? где это уравнение состояния выходит за единицу вне погрешности?
124 490331
>>90330
Как оно в 1980-х? Ускоренное расширение вселенной уже подарило пару нобелевских еще в нулевых же.
Раз раньше расширялась медленнее, чем сейчас, можно сделать далеко идущий вывод, что и дальше будет расширение с ускорением. И, значит, может настать момент, когда гравитация не удержит.
коты.webm7,4 Мб, webm,
640x360, 1:08
125 490332
>>90328
Бамп вопросу.
126 490342
>>90313
Если уебаны обладают технологиями, позволяющими им прилететь пару сотен световых лет и все на месте все колонизировать/сожрать/экстерминировпть - то они уже давно должны знать. Если не про нас конкретно, то про жизнь на Земле вообще.

Риск то конечно есть все равно, но он намного меньше риска того что мы самоуничтожимся раньше. Предположительно.
127 490344
>>90328

>Как (какими способами) вообще можно собирать межзвездный водород, если корабль летит на релятивистских скоростях?


Никакими.

>Как оценить до какой скорости будет эффективен термоядерный двигатель?


До тысяч км/с

>Что там со скоростью истечения, будет ли излучение вносить заметный вклад в тягу?


Нет.

>И если межзвездный водород хорош для самой реакции,


А если бы у бабушки был хуй.
128 490352
>>90344
Добра тебе, няш.
Получается вариантов с подзаправкой на ходу пока нет. Жаль, придется тащить все на себе. Хотя это все влажные, мы и свою систему пока никак не обживем.
129 490353
>>90352

>тащить все на себе


Были расчеты намного старше тебя - чтобы пролететь смешные сотни с/л со смешными долями с и при этом тащить все с собой - стартовая масса начинается от миллиардов тонн.
На аннигиляции это все, естественно.
130 490354
>>90353
В том-то и хуйня тащем-то. Я чо спрашивал-то про дозаправку?
131 490356
>>90354

>Я чо спрашивал-то про дозаправку?


Потому что наивный дурачок, очевидно же.
IMG0205.PNG930 Кб, 1024x768
132 490357
Почему Солнце только вошло в Близнецы, а уже по всем гороскопам наступил Рак?
pistol-pencil.png15 Кб, 400x300
133 490358
>>90356
Да вы тут совсем охуели. Ну ок, мяукни еще что-нибудь.
134 490359
>>90328
нуу, был такой концепт Bussard Ramjet, создание магнитной воронки в несколько км для засасывания водорода на ходу, однако дальнейшие расчеты показали что "кпд" будет отрицательным и эта хуйня будет только тормозить корабль. Впрочем, торможение тоже важная часть полета.

>Как оценить до какой скорости будет эффективен термоядерный двигатель?


по классике - скорость истечения рабочего тела умножить на 2 или 3 примерно

>Что там со скоростью истечения, будет ли излучение вносить заметный вклад в тягу?


не

>можно ведь домешать туда что-то потяжелее, для увеличения импульса


вообще-то чем легче выхлоп тем выше импульс удельный
есть варианты с добавлением термоядерного топлива прямиком из реактора, дейтреий, тритий, гелий3 и прочее такое, их можно с собой брать
135 490361
>>90358
Вернись в обо/сцай, котомальчик.
136 490363
>>90359

>Bussard Ramjet


Да, вот смутные воспоминания были, попозже обмажусь.

>импульс удельный


Удельный. Но водород в баках возить такое себе.

>дейтреий, тритий, гелий3


Добра тебе. А если с другой стороны смотреть, как можно увеличить скорость истечения? Можно ли играть с конфигурацией магнитной ловушки, или еще с чем, чтобы поднять? Ну или например доразогнать магнитным полем дополнительно, а запитать с того же термояда разгон?
>>90361
Я тут который тред. Просто тут анон еще походу помнит, чем тематика от /b/ отличалась.
137 490365
>>90363

смотри разные варианты термоядерных двигателей, их дохуя
138 490367
>>90342
а если пришельцы решат, что это люди это уебаны, которые будут угрожать им в будущем? Тогда они могут решить избавиться от уебанов, которые могут в будущем добраться до них. Тебе сложно осознать какие же мразные выродки эти люди, так как ты сам человек. Но пришельцы столкнувшись с людьми могут почувствовать угрозу, даже если это угроза будет для них в далеком будущем. Это хуже чем смерть звезды через несколько миллиардов лет - миллиард это немыслимо, но цивилизации быстрее, уже за тысячу лет человечество может уйти очень далеко. Ты представь если бы мы засекли цивилизацию мерзких плотоядных жуков в 200 световых годах от нас. И еще мы засекли бы, что они очень воинственные, воюют между собой постоянно. Вполне можно было бы предположить, что через 1000 лет они бы смогли развить технологию и добраться до нас. Если бы мы были зациклены на безопасности, то мы могли бы попытаться обезопасить себя, попытавшись либо уйти подальше от них, либо применив какие-то другие шаги? Можно многое ожидать от нас, и от пришельцев.
139 490371
>>90367

>уже за тысячу лет человечество гарантированно скатится и выродится


Поправил эту мартышку.
140 490384
>>90367
Это вполне легитимное опасение, но только большинство учёных и прочих серьезных дядек и тетек прилагавших руку к отправкам сообщений в космос идеалисты-романтики и так не думают.

Ну и опять же. Даже если допустить, что наши радиопередачи не детектируются на расстоянии больше нескольких световых лет, то Земля сама по себе это сильно необычная планета, которую заметно издалека. Если цивилизация, как и мы, искала себе в космосе "второй дом", и нам "посчастливилось" иметь схожие критерии жизнепригодности - нашу планету давно обнаружили. Мы и сами уже приближаемся к этой возможности, осталось только дождаться нового поколения телескопов вроде TESS или ATLAST.
141 490386
>>90265
>>90282
А в шапке есть про это вопрос, лол.
Да, сам не пролезет. Если за минуту окажется в шлюзе - может и можно будет откачать.
142 490387
>>90344

>До тысяч км/с


Почему так?
Почему нельзя ускоряться сколько влезет?
143 490389
Пацаны, а наша Земляшка реально может перевернуться в ближайшем будущем?
144 490390
>>90389
Нет.
Магнитные полюса могут поменяться, но это будет не в одночасье.
145 490391
>>90389
Хотя если ты про эффект Джанибекова - то нет.
Он к полностью симметричным объектам вроде сфероидов не применяется, надо чтобы было три разных момента инерции.
146 490397
>>90389
Ващет каждый день переворачивается, если ты не заметил.
147 490400
>>90391
Да, я про это. Спасибо за ответ.
148 490404
>>90391
Так, стоп. У нас же Земля и не сфероид полностью симметричный!
149 490409
>>90404
Так Земля и не вокруг промежуточной оси вращается.
150 490413
Предположим, расширение Вселенной стало быстрым, но не ускоряется (чтобы не было Большого Разрыва жопы). Скажем, за год любое расстояние увеличивается вдвое. При таком расширении, как я понимаю, мы даже соседних звезд не увидим, пушо они будут удаляться быстрее скорости света, не то что Галактику.

А что будет с Солнечной системой? У нее размеры вроде такие, что гравитация будет легко компенсировать это расширение. А орбиты планет дестабилизируются? И если да, то как - хаотично, или как-то предсказуемо, скажем, будет увеличиваться среднее расстояние от Солнца или эксцентриситет?
151 490416
>>90404
For all intents and purposes - симметричный. Вот комета Чурюмого Герасима вполне могла бы менять ось.
152 490422
>>89743
Начнут мусор перерабатывать, космос никогда не будет выгоден.
153 490423
>>90357
Потому что современные астрологи в астрономически наблюдения не умеют и пользуются данными тысячелетней давности, а с тех пор карты звездного неба сильно изменились.
154 490424
>>90423
Ты еще спроси как называется столица рептилоидов на Нибиру.
155 490430
>>90424
Ее название изменилось?
156 490437
>>90424
Москва?
157 490458
>>90413

> Скажем, за год любое расстояние увеличивается вдвое.


Ниет. Это так не работает.

>При таком расширении,


Нет, пусть другим будет.

>А что будет с Солнечной системой?


Ну я так думаю: Надо прикидывать это твое расширение, например для двух тел, Солнце-планета. Посчитать на сколько разлетятся за интересующий период, оценить насколько это меньше r (расстояния от Солнца). Если намного меньше - вряд ли будут большие пертурбации, по крайней мере сразу.

>А орбиты планет дестабилизируются? И если да, то как - хаотично, или как-то предсказуемо


Чот мне кажется, что при достаточно больших значениях расширения будет обратно пропорционально массам, чем меньше - тем быстрее будет "улетать". Мне представляется это как плавный зум на интерактивной карте солнечной системы, причем орбиты более тяжелых планет при этом будут "увеличиваться" более плавно.
158 490468
>>90458
По последним данным, "постоянная" Хаббла сейчас таки больше, чем в эпоху возникновения реликтового фона, так что разрыв жопы или еще какая неведомая хуимбола вполне может случиться со вселенной за космологически небольшое время, когда гравитационно связанные системы еще будут существовать.
159 490606
>>90391

>Хотя если ты про эффект Джанибекова - то нет.


>Он к полностью симметричным объектам вроде сфероидов не применяется


А в самом примере эффекта Джанибекова объекты симметричны. Как минимум по двум осям
https://www.youtube.com/watch?v=agEn8M5SM_o
Так-то и Земля асимметрична - северное и южной полушарие ну совсем не одинаковые, куда более неодинакорые, чем, скажем, восточное и западное
160 490611
>>90458

>Ниет. Это так не работает.


Да? А как это работает?
161 490661
>>90468>>90458
Насчёт ускоренного расширения Вселенной и постоянной Хаббла - это всё пока очень условная экстраполяция с применением лютых костылей.

Например, нет никаких гарантий, что она не стационарна на очень больших масштабах, а на "малых" (которые могут превышать весь космологический горизонт) не квазистационарна - т.е. когда одни фрагменты расширяются, другие сужаются, и так флуктуируют. Поскольку взаимодействуют сами куски пространства - здесь запросто может не работать принцип причинности. А замерить красное смещение мы в любом случае можем только из одной точки.

Например, есть теория, что весь Млечный путь и вся местная группа находятся в середине невъебенных размеров войда (т.н. войд КВС), который настолько огромный и рыхлый, что заметить его изнутри достаточно сложно. А если это так, то постоянная Хаббла у нас выше из-за банального гравитационного притяжения к стенкам этого войда.
162 490672
Можно с земляшки сфоткать плутон так же пиздато как нюхарайзенс?
163 490679
>>90661

>есть теория


А еще говорят, что кур доят.

>может не работать принцип причинности


Не слишком ли дорогая цена для объяснения небольших фактов?

>находятся в середине невъебенных размеров войда


И опять-таки противоречие с базовым принципом изотропности. А разгадка проста: так называемые "космологи" это народ-пидар, народ-мразь, народ-говно, народ-гнида, народ-гной, народ-хуесос.
СМЕРТЬ ВАНЮЧЕЙ КОСМОЛОГУСНЕ!!!
164 490686
>>90672
С поверхности точно нет, атмосфера не позволит, с орбиты хуй знает, если телескоп будет достаточно большим, то с хорошей выдержкой может что-то подобное и получится.
165 490690
Бывает такое, чтобы планеты оказывались буквально на одной прямой, типа Марс с земли проходящий строго перед Юпитером, или Юпитер перекрывающий вид на Твойанус?
Или они так неровно на эклиптике что такое при нашей жизни не случится?
166 490691
>>90690
да, гугли парад планет
167 490693
>>90691
Это ДРУГОЕ, они не на одной линии, они на одной плоскости просто, это хуйня. Хочу затмение сатурна венерой.
168 490731
>>90690
Бывает, но охуенно редко. Даже прохождение по диску Солнца-то нечастое событие, а оно в сравнении с другими планетами здоровенное.

https://en.wikipedia.org/wiki/Planetary_transits_and_occultations

Следующий раз будет 22 ноября 2065 – Венера перекроет Юпитер.
169 490746
Из аккреционного диска что-то может собраться?
170 490747
>>90331

> как


как оно до изобретения письменности?
прочитал бы уже что большой разрыв и приращение длины метрики на 6% за мульярд лет(которое и ускорение и рывок и прочие производные показательной функции) разные понятия, сектантик
дарю тебе открытие, что нобелевки относятся исключительно к несвязанным гравитацией обьектам
171 490748
>>90133
Почему звезда HD 140283 с большей массой, чем Солнце, живет дольше? Солнце превратится в красного гиганта через 7,5 млрд лет (12 млрд, если считать с возникновения).
sage 172 490749
>>90747
Бля, ахуительная история!
173 490750
>>90679

>А еще говорят, что кур доят.


Все эти большие разрывы и большие сжатия - из той же серии.

>Не слишком ли дорогая цена для объяснения небольших фактов?


Ну здрасте, у тебя как минимум прямо сейчас видимые объекты по разные стороны космологического горизонта друг относительно друга со сверхсветовой движутся.
А касательно дорогой цены для объяснения небольших фактов - это к теоретикам тёмной энергии.

>И опять-таки противоречие с базовым принципом изотропности.


Ты нахуя тащишь в космологию принцип из ньютоновской механики? Он уже как минимум на планковских размерностях и в чёрных дырах не работает.
174 490752
>>90468
ты сейчас большой взрыв опровёрг так-то
если взять текущий параметр хаббла - в 6-7% за миллиард лет - то за возраст вселенной оная увеличилась в единицы раз, если брать, как ты, меньше, то от БВ совсем нихуя не останется
175 490755
>>90750
Маня? Маня.
За темную энергию в простейшем случае ответственен блять еще более базовый член в лагранжиане Эйнштейна-Гильберта, чем тот, который ответствен собсна за гравитацию. Так что просто иди нахуй.

>на планковских размерностях и в чёрных дырах не работает


Шо несет, вообще ахуеть. Ты тот ебанутый кефирщик, что регулярно засирает спейсач своими шизоидными высерами, да?
176 490758
>>90749
современная космология она такая, братиш
177 490760
>>90748
металичность катализирует ядерные циклы
а она из второго поколения ещё
178 490779
Почему гравитация не разрывает едро во все стороны?
image.png10 Кб, 793x46
179 490784
>>90746
А сам-то как думаешь?
180 490785
>>90779
Чтобы что-то разорвать нужен градиент сил, чтоб суммарно в разные стороны тянуло.
А тут в ядре в какую точку ни посмотришь - все стремится строго в центр, а в центре уже суммарно никуда не тянет.
181 490814
Такого рода бои в космосе вообще возможны?
https://www.youtube.com/watch?v=p-6TaGGqZUQ
182 490817
>>90814
Стрелять в космосе можно, но стрелка будет пидорасить во все стороны. Проще лазером прожечь дырку в скафандре.
183 490824
>>90817
какая еще стрелка?
184 490830
>>90824
Того который стреляет. Тебя будет мотать, отбрасывать, закручивать.
Это как на Земле драться на детонационных двигателях.
185 490832
>>90814
>>90817
ну, там видны двигатели на скафандрах, очевидно они компенсируют импульс оружия
неясно только нахуй им вообще классический огнестрел когда можно просто стрелять мелкой дробью и гранатами с дистанционным подрывом
186 490833
>>90357
потому что все гороскопы устарели на 3000 лет
https://spaceplace.nasa.gov/starfinder2/en/
187 490834
>>90832
А почему бы и не огнестрел?
Энергетическая емкость отличная.
Отдачу можно компенсировать отводом пороховых газов, теряя в мощности.
Нужен безгильзовый патрон, на самом деле, т.к. флудить гильзами на орбите себе дороже.
188 490835
>>90834
Или гильзосборник, двойной магазин, например.
189 490839
>>90832

> когда можно просто стрелять мелкой дробью


Один из классических космосрачей
190 490880
Я так понял тень чёрной дыры аккуратно размером с фотонную сферу. Но это с источником излучения вне этой сферы. Но что с излучением падающим между ФС и ГС. Да, понятное дело, что вещество там долго не задерживается, но так или иначе существует и излучает. Правильно я понимаю что если фотография будет более чёткой, то можно будет видеть тень уже ГС за счёт вещества находящегося ниже ФС? Насколько, если это возможно, надо увеличить детализацию дабы обнаружить это?
191 490889
>>90079

> довольно сильное отличие.


Если ты – гуманитарий.
192 490890
>>90367
Ты только что "Задачу трёх тел".
193 490909
>>90069
Нет, не вытекает. Больше того, в СТО нет никаких прямых ограничений на скорость движения.

Теория относительности вытекает из совершенно базовых геометрических аксиом и фактов, наблюдаемых без строгих экспериментов, и ни из чего больше. Она буквально и появилась таким образом.
194 490942
>>90909

>Теория относительности


Специальная - уточняй всегда.
195 490952
>>90909

>совершенно базовых геометрических аксиом и фактов, наблюдаемых без строгих экспериментов


Ща погуглил, основной постулат там то, что скорость света в вакууме одинакова в любой инерциальной системе отсчета. Экспериментальное подтверждение этого все же нетривиально и сам факт неочевиден.
196 490995
>>90952

>скорость света в вакууме одинакова в любой инерциальной системе отсчета


Вообще-то это следствие.
Ты какой-то тупой, из роскосого треда наверняка.
197 491006
>>90995
Ага. Следствие чего?
198 491009
>>91006
Сигнатуры Минковского.
Еще дебильные вопросы будут?
199 491011
>>90942
Верно
200 491027
>>91009
А, ясно, это просто аксиома Минковского, очевидный базис теории.
201 491060
Возможно ли вычислить координаты эпицентра Большого Взрыва?
202 491068
>>91027

>аксиома


Все же сигнатура.
Там есть два варианта - и подтверждается только один, а второй дает жутчайший шизобред, такой даже местным шизикам из /мг/сн/зог не снился.
Да что там - такой бред даже мне не снился. Наверно.
203 491076
>>91060
Невозможно вычислить то, чего не существует.

Большой Взрыв и последующее расширение Вселенной начались во всех точках пространства одновременно.
204 491080
>>91009
https://en.wikipedia.org/wiki/Postulates_of_special_relativity
Интересно, а это, видимо, дебильные википидоры написали.
205 491085
>>91080
Вовсе нет, это дебильный червь-питурд (ты) прочитал
206 491093
>>91085
Я понял, постулат не постулат, а следует из сигнатуры, а я дебил. Спасибо, что разжевал.
207 491108
>>90080
очень похоже на YF-23
208 491121
>>91093

>а я дебил.


Золотые слова.
Удачи.
209 491164
Куда расширяется вселенная?
210 491165
>>91164
во все стороны
211 491167
>>91165
Ну это понятно. А что она заполняет? Вот вселенная состоит из пространства. Во что расширяется это пространство?
212 491168
>>91167
само в себя
важно понимать что это не как надувание шарика, когда расширение идет от центра, а по другому, расширение идет из каждой точки пространства, каждой элементарной частицы, во все стороны относительно других точек пространства
213 491174
214 491175
>>91174

>the strength of the signal reaching Earth has been described as 1,000 times less than from a mobile phone on the Moon


Чето пежня какая-то
215 491185
>>91174
Про какую штуку, /news/ или FRB? Первое - раковник который надо сжечь и вход завалить камнем, второе - не объяснённое пока явление, к разгадке которого понемногу приближаются. Горячая тема в астрофизике, которую еще недавно принимали за наземные помехи радиотелескопам. Локализация FRB это один из первых шагов к объяснению. По скоротечности и интенсивности сигнала понятно, что его генерирует какое-то катаклизмическое событие, но механизм пока никто не объяснил. Есть несколько гипотез, но пока еще слишком рано говорить о чем-либо.
216 491208
>>91168
Так Вселенная имеет форму тора или бутылки Клейна?
217 491209
какая частица сейчас считается самой маленькой и неделимой - нейтрино или кварки?
StandardModelofElementaryParticlesru.svg.png317 Кб, 1280x961
218 491219
>>91209
Смотря что ты понимаешь под словами "самой маленькой и неделимой".

Ни одна из частиц на картинке не делятся ни на какие известные современной науке "составные части". (Преоны - это только гипотеза; точнее, сразу куча разных противоречащих друг другу гипотез.) В этом смысле они все в равной степени "самые неделимые".
Если "самые маленькие" в смысле массы - массы покоя у фотона и глюона равны нулю. Меньшие массы чего бы то ни было неизвестны. Хотя в рамках гипотезы - масса может быть отрицательной и даже мнимой (у тахионов).
219 491220
>>91219

> Ни одна из частиц на картинке не делятся ни на какие известные современной науке "составные части".


P. S. Ну и там не показаны ещё соответствующие им античастицы: для электрона - позитрон и др.
220 491221
бамп
ПеКа-фейсы-макросы-why-so-butthurt-auto-162334.png13 Кб, 734x748
221 491240
>>91219
РОДИЛСЯ КВАРКОМ
@

СТРАННЫМ
image.png663 Кб, 780x910
222 491241
>>91219
Поясните за бозон Хиггса и разницу W и Z.
Глюон "клеит" кварки в протонах и нейтронах, сильное взаимодействие.
Как выражается слабое и что делает Хиггс?
223 491273
Лежу засыпаю и слушаю, как попов говорит, что дыра не интересный объект и вращается.
Там есть чему-то вращаться в виде самого объекта, которого нет, а не веществу вокруг горизонта?
224 491275
>>91219

>Смотря что ты понимаешь под словами "самой маленькой и неделимой".


в прямом смысле самый маленький объект?
Есть же квантовая пена
Что сейчас самое маленькое из известного во Вселенной?
225 491276
Как вы относитесь к Теории бесконечной вложенности материи - https://ru.wikipedia.org/wiki/Бесконечная_вложенность_материи ??
226 491278
227 491286
Есть ли моря на Венере?
Я немножко аутист, но:
- во многих источниках указываются осадки из концентрированной серной кислоты
- t кипения H₂SO₄ около 340 градусов по этому вашему Цельсию при давлении в 1 атм.
- на Венере, если верить тем же источникам, давление где-то 90 атм, а температура порядка 460 градусов.
- помня из школьного курса физики, что чем выше давление, тем выше температура кипения, можно предположить, что температура кипения серной кислоты на Венере гораздо выше, чем на Земле
- из этого следует, что осадки должны собираться в достаточно крупные водоёмы
Есть какие-то данные по этому вопросу?
228 491288
Браузерная игра на 10 минут про поселение человечества на другой планете (важно правильно выбрать планету):
http://philome.la/johnayliff/seedship/play
Произойти может очень много всего разного, по игре есть целая фанатская вики.
229 491289
>>91286
Там СО2 в виде сверхкритической жидкости. Где-то в низинах.
Водой там даже не пахнет.
Серьёзно, не нюхай.
230 491296
>>91286
Емнип серная кислота там только в верхних слоях атмосферы и ее крайне мало, про подобные осадки первый раз слышу.
231 491317
Про какие пиздецы еще можно почитать? Типа чёрные дыры, гамма-всплески, магнетары - люблю такие йобы.
232 491368
>>91289

>в виде сверхкритической жидкости


Да, имею в виду именно какое-нибудь подобие жидкости, а не конкретно H2O. На Титане, например, жидкие метановые озёра при -170. Почему-бы не быть серным озёрам на Венере.
233 491434
>>89400 (OP)
САП учоные!

Недавно посмотрел кино о реликтовом излучении -- https://www.youtube.com/watch?v=Q655Siyo9h4

Там этот фраер, Олег Верходанов стелит так складно, шо почитать чтобы так же пиздеть? Надеюсь вопрос достаточно тупой для етава треда
234 491438
>>89780
А представьте, во сколько миллиардов раз больше СТРАДАНИЙ, если есть миллиарды населенных животиной планет как Земля, которые жрут друг друга, и человекообразных живодеров. Дануна
235 491440
>>91434
А тибе чо - реторику или научные данные ыыы?
236 491445
>>91440
НАУКА, ну там всяких интересностей тип физики атмосферы звезд и прочих физик
237 491446
>>91445
Нахуй иди, значит.
238 491449
>>91445
нет.
239 491450
>>91445
Почитай это - очень простым языком объясняется
http://nuclphys.sinp.msu.ru/pilgrims/index.html
240 491451
>>91450

>М.И. Панасюк


А без хохлов можно?
242 491457
>>91454
Угомонись, хохол.
Крым чей?
243 491467
>>89827
Очень жизненная аналогия, спасибо
244 491468
Как нужно праздновать 50-летие ступания на Луну человечка?
sage 245 491477
>>91468
Сходить в темное время суток в безлюдное место и, бормоча вполголоса отрывок из Некрономикона, как он приведен в лавкравтовском Ужасе Данвича, обосраться.
246 491478
>>90282

> лимит армрестлинга


)))))
247 491484
>>90314

> Мысль, что мы одни, как минимум в пределах взаимодействий со скоростью меньше С и сроками не более тысячи лет - пугает больше.


Зато хоть не будем СТРАДАНИЯМИ в других мирах головы забивать, будем над своими проблемами думать
248 491486
>>90361
Обо/sci же
249 491552
На Церере есть рачки?
250 491562
>>91552
Только жуки.
251 491569
>>90686
Достаточно большим - это 10 км в длину?
252 491570
>>90750

> видимые объекты по разные стороны космологического горизонта друг относительно друга со сверхсветовой движутся.


И что, это по-твоему нарушает казуальность?
image.jpeg130 Кб, 604x300
253 491574
>>91451
Тебе чем-то не нравится Понас?
254 491594
>>91570
Ты кого казуалом назвал, эй
255 491596
>>90672
Нет, нельзя, нужен телескоп в дохуя сотен км апертурой.
256 491662
Я залетный и у меня действительно глупый вопрос. Я от нечего делать и от хорошей погоды где-то неделю ночью наблюдаю за звездами. Видимость очень хорошая, безоблачно.

Так вот, видел все это время очень яркую, не меняющую позиции мерцающую звезду, которую не смог найти ни на какой карте. A вот вчера я ее больше не увидел и так и не понял, что это могло быть. Какие могут быть варианты? Может какая планета? Инб4: НЛО
257 491664
>>91219

>(Преоны - это только гипотеза; точнее, сразу куча разных противоречащих друг другу гипотез.)



>Впрочем, история с адронами и кварками, а также симметрия между кварками и лептонами, наводит на подозрение, что кварки могут сами состоять из чего-то более простого. Рабочее название для гипотетических частиц-составляющих кварков — преоны.



>Преоны — гипотетические элементарные частицы, из которых могут состоять кварки и лептоны. Несмотря на то, что на сегодняшний момент нет пока никаких экспериментальных указаний на неточечность кварков и лептонов, ряд соображений (наличие трёх поколений фермионов, наличие трёх цветов кварков, симметрия между кварками и лептонами) указывает на то, что они могут быть составными частицами.

258 491667
>>91664
И что ты хочешь сказать этой цитатой?

В Википедии ровно то же написано, что и я написал. Что пока дальше интересных гипотез дело не продвинулось.
259 491694
>>91662
Карты, надеюсь, были на тилибоне? Юзай это https://apps.apple.com/ru/app/star-chart/id345542655
260 491697
>>91273
Да. Момент инерции должен сохраняться, потому что законы сохранения.
261 491700
>>91694
Спасибо, на ведре нашел.
1561593113498.jpg137 Кб, 1080x1350
262 491834
Какую рациональную идею можно предложить для миссии к Церере? Прост уже сейчас понятно, что дискурс структурировали таким образом, что идет цепочка "сначала Луна, затем Марс". А как можно повлиять на общественный диалог, чтобы вставить в эту схему Цереру либо перед Марсом, либо после Марса? Потому что вдруг люди слетают на Марс, сдохнут там или вернутся, а потом наступит стагнация и нежеление лететь куда бы то ни было.

Церера выглядит как-то скучновато, или можно как-то заинтересовать ею людей? Или же выбрать для этих целей Весту, или Каллисто?
263 491843
>>91834
Говно без задач, тащемта. Меньше Луны, дальше Марса. Нет атмосферы, нет подповерхностной жидкой воды, насколько известно.
264 491845
>>91843
Ну, это на первый взгляд действительно так, согласен. Но это никак не поможет нам. Представь что ты маркетолог и тебе нужно продать это говно без задач быдлу, чтобы они хотели туда слетать. Если мы не сможем это сделать, то человечество будет обречено торчать на Земле вечно пока технологии виртуальной реальности не разовьются настолько хорошо, что все люди забьют хуй на космос и отправятся няшить 2D шлюх в виртуальный Оазис.
265 491846
>>91834
Да никаких причин нет туда лететь, Церера это полная параша, хуже, чем Луна, хуже, чем Марс. Интересного там нет абсолютно ничего.

Реально интересно было бы слетать, скажем, к Психее. Такой-то огромный кусок металла вместо унылого бесполезного камня, и поизучать есть что, и в далеком будущем местные ресурсы поосваивать.
crossing.jpg146 Кб, 800x600
266 491847
>>91845

>Представь что ты маркетолог


А теперь вытащи маркетинговый хуй в форме бутылки от кока-колы © из ануса. Быдло ты не уговоришь хотеть в науку, одновременно пихая им во все дыры потреблядство. По твоей логике людечество и обречено жить на Грязи вечно, захотят в космос ровно за пять минут до начала акопалипсиса, но будет немного поздно. Тут надо подходить разумно, чтобы расходы намногоплатильщиков не вызывали бурления говн, а цели можно было донести до зайчатков быдла попродвинутей. А когда привыкнут, что люди летают/живут на Муне/Дюне уже можно дальше щупать небесные тела.
267 491849
>>91846

>Такой-то огромный кусок металла вместо унылого бесполезного камня


Pssst, у меня плохие новости. Психея могла быть частично дифференциирована, так что на поверхности там скучный унылый бесполезный камень как на Весте с парой гигантских кратеров, в центре есть металлическое ядро, но оно скрыто и застывшее.

>>91847
Надо заранее думать над тем чтобы сделать все тела солнечной системы интересными для быдла. Можно даже мемы использовать для этого. Помню Баз Олдрин пускал утки про монолит на Фобосе, чтобы хоть как-то заинтересовать людей в миссии к этому объекту.
https://en.wikipedia.org/wiki/Phobos_monolith
Нам нужно также думать - даже если тело кусок говна, надо смотреть на него с лупой, с микроскопом, находить на нём интересный камень и форсить его. Ученые уже делают это, но они используют стратегию "поиск жизни" постоянно пуская утку "а что если на теле Х есть жизнь? нужно отправиться туда и исследовать!". Но думаю это не единственная модель пробуждения интереса.
268 491850
>>91849

>Pssst, у меня плохие новости. Психея могла быть частично дифференциирована, так что на поверхности там скучный унылый бесполезный камень как на Весте с парой гигантских кратеров, в центре есть металлическое ядро, но оно скрыто и застывшее.



С каменной поверхностью альбедо бы гораздо меньше было, радаром уже сканировали и выяснили, что 90% поверхности это металл.

https://www.naic.edu/ao/blog/radar-observations-and-shape-model-asteroid-16-psyche
https://en.wikipedia.org/wiki/16_Psyche

>Composition and origin


>Radar observations indicate that Psyche has a fairly pure iron–nickel composition, consistent with it having one of the highest radar albedos in the asteroid belt (0.37±0.09). Psyche seems to have a surface that is 90% metallic (iron), with small amounts – 6±1% – of orthopyroxene.

MirKolco.jpg209 Кб, 1280x720
269 491852
>>91849

>Надо заранее думать над тем чтобы сделать все тела солнечной системы интересными для быдла.


Когда случайно заходил разговор про освоение космоса рядом с нормисами, те обычно кривились, как будто муху съели только что с говна слетевшую. Им не интересно, им не интересно что происходит в стране рядом, что происходит в своей стране, максимум свой город и свои друзяшки из соц-паутин. И им не будет интересно, потому что их внимание, их фокус, может удержать только яркий клип не длиннее 5 минут. Так что они по определению якорь, а не винт. Двигать должны другие, с этих только бабло в виде налогов, желательно под легким сми-наркозом.
270 491854
>>91850
не металл, а металлосодержащие минералы. Но я вот что скажу - мне нет причин разочаровывать тебя, когда я пытаюсь как раз выдумать обратное - идеи как заинтересовать людей в космосе. Пусть будет металлический астероид, посмотрим что из этого выйдет. Надеюсь на поверхности найдут какие-нибудь металлические кристаллы, зеркала, шпили, монолиты.
271 491856
>>91852
Так освоение космоса это как третий этаж, а здание не строят с третьего этажа. Сначала нужно людей заинтересовать в астрономии, космонавтике, а уже потом на более сложные уровни выходить - начинать про освоение космических тел. Большинство людей даже не поймет что такое Церера, Психея для них это какая-то говнарская группа.
В России сейчас появились парочка годных научпоп каналов, каналы астрономов и университетов. Это уже что-то, но мы можем сделать больше. Не нужно надеяться, что американцы всё сделают первыми. Космизм придумали в России, а не в США.
Dyson-swarm.png226 Кб, 1016x622
272 491860
>>91856
Я тебе только что расписал круг интересов нормиса-потребляциониста. Туда даже своя страна не входит, и не войдет, пока такая система ценностей. Так-то оно и не больно надо, по сути, ничего полезного они внести не могут как и мы тут, так что самое значимое что у них нас есть - это кошельки, с которых налоги, с которых ракеты. Но даже это не про нашу страну, тащем-то.
yuki-1.jpg69 Кб, 796x637
273 491866
>>91860
Есть лазейки в их систему ценностей и круг интересов. Вот взять например чувака, который работает на Forklift или вилочном погрузчике - его мог бы заинтересовать робот погрузчик из чужих, или устройство вилочного погрузчика на Луне, который мы могли бы туда отправить для создания лунной базы. Нужно находить такие вот лазейки в мир простых людей. Какой-нибудь садовник вполне может заинтересоваться тем какие травы хотят выращивать колонисты на Луне в оранжерее. Хотя казалось бы - где садовник, и где Луна! Связь эфемерная, но она позволяет приземлить далёкий мир и соединить его с интересами простых людей, которых мы пренебрежительно назвали бы просто быдлом. Нужно не строить эзотерику, вот что главное. Чем проще, тем лучше.
274 491891
>>91834

> церера


никак

> каллисто


лучше луны и марса по
радиации(считай совсем нет)
лучше луны по
суткам(16,5 вместо 29,5)
а ещё она няшная и нордическая
ну вот плюсы за которые можно топить и закончились
18578549091206d15c9f5a949ed7cf95.jpg3 Мб, 1182x723
275 491893
>>91866
Так согласен, чисто номинально приобщать, чтобы не было отторжения. Вплоть до магнитиков из космических камней Это минералы!. Тем более что космос нам уже подарил много удобных мелочей.
276 491904
>>91891
На Церере есть Кратер Оккатор и Гора Ахуна
https://en.wikipedia.org/wiki/Ahuna_Mons

На Каллисто еще вот это есть - мелкогорье с белыми шапками, по английски knobs называют
https://www.youtube.com/watch?v=LILgPtbQ8ds
Круговые ударные структуры еще, если их датировать то можно многое узнать о времени и истории в Солнечной системе. Ну и отправиться в Вальхаллу и сесть туда не метафорически, а реально довольно круто
Есть кратер Лофн, который как Оккатор, только на стероидах.
Подлёдный океан еще
Можно с Каллисто смотреть за другими спутниками например, как астрономический пост там организовать
277 492001
>>91834

>Какую рациональную идею можно предложить для миссии к Церере?


Очевидная перевалочная база для добычи редких минералов на астероидах. Но это точно не перед Марсом, и сперва туда в любом случчае надо отправить целый флот роботов-геологоразведчиков.

>Или же выбрать для этих целей Весту


Веста это Церера на минималках.

>или Каллисто?


Ты долети до неё сначала. И съебать оттуда тоже непросто, чтобы и гравитационной ямой Юпитера воспользоваться, и не убиться об его радиационные пояса.
sage 278 492021
>>92001

> заинтересовать


> Ты долети до неё сначала

279 492029
>>92021
Заинтересовать и Альфой Центавра можно, фигли толку-то.
Юпитер в 4 раза дальше, чем Марс. Летать туда на химии - за пределами технологий и человеческих возможностей. Здесь без вариантов либо ЯРД, либо ТЯРД, либо плазменный двигатель, запитанный от реактора мегаваттного класса, а это всё пока очень сырые технологии. Плюс ещё есть всякие петушиные лобби дегенератов, у которых от слова "радиация" говно из ушей лезет.
OrionTaylor.jpg102 Кб, 1100x1284
280 492030
>>92029

>Юпитер в 4 раза дальше, чем Марс.


Давно деды всё придумали.
281 492035
>>92001

>это точно не перед Марсом


А почему бы и нет? На Цереру проще высадиться, там проще добыть воду путем ИСРУ, проще улететь оттуда. Можно потренироваться выживанию в долгой миссии.
Ученым может быть интересно взять образцы белой кончи - смотри какая мякотка. Этот файл такой подробный, что даже на спейсач не залить.
https://upload.wikimedia.org/wikipedia/commons/5/58/PIA21924_-_Mosaic_of_Cerealia_Facula_in_Occator.jpg
Представь как круто сесть на это дерьмо, какой вид откроется во все стороны. это вам не KSP где "всего два склона, камень, делай флаговтык и пиздуй отсюда", это реальное место с кучей камней, истории, можно поездить на машинке, пофоткать, развернуть микроскоп на месте, лазер, взорвать что-нибудь, растопить...
282 492169
Насколько вероятно, что еще при нас, то-есть в ближайшие 50 лет человек высадится на Марсе? Все время слышу что мол "скоро" и "вот-вот собираются", но мне что-то кажется, что это какой-то пиздеж и попил бабла.
283 492170
>>92169
Ну сразу после летающего автомобиля и прямо перед коммерческим термоядом.
284 492207
>>92170

> сразу после коммерческого термояда


Пофиксил тебя
285 492217
В чём смысл постоянно ездить по планете в охоте за затмениями? Ну увидел разок, прикольно - но и не более. На ютубе глянуть можно, в конце концов.Неужели оно того стоит?
286 492225
>>92217

>в чем смысл гоняться за тнями, просто подрочил на порнхабе и норм


Трустори
287 492227
>>92207

>до/после коммерческого термояда


Чот постиронично похихикал.
>>92217
Нищета и огороженность формирует другие ценности. Если была бы возможность спокойно летать по шарику - почему нет? Весь мир открыт, лети куда хочешь, цена вопроса 3-5к рублей. Ну не нравятся затмения - можно найти что-то другое интересное. Даже какой-то процент сычей тоже вылезали бы, если бы выходя в подъезд не окнлсь в атмсфр фллт с ннткй бзсхдн.
288 492273
Если из телескопа убрать корпус и оставить только линзы на нужном расстоянии, то будет ли он по прежнему функционировать?
289 492275
>>92273
Все молчат. Предположу, что будет, но появятся блики и засветка от рассеяного света сбоку.
291 492279
>>92273

>19:19:25


>>92275

>19:52:17


>Все молчат


Соси хуй, быдло.
292 492280
>>92279
Слоупочине неприятно? Ну давай поглажу волосы, покурлыкаю, курлы-курлы. Ты такой няшик, когда злишься и косплеишь тупого быдлана. :3
293 492282
>>92280

>косплеишь


Давно ты в ебасос не получал, смотрю.
294 492284
>>92282
Ты злой, потому что голодный. Братишка, я тебе покушать принес!
295 492285
>>92284

>потому что голодный


Уже нет, сожрал полкурицы и чай.
15535994510110.jpg36 Кб, 604x453
296 492288
Как должна быть устроена планета/иное космическое тело, чтобы обладать массой, обладать гравитацией, но при этом не иметь ускорения свободного падения?
Это вообще возможно?
Если да, то как?
Если у нас будет планета-мяч, где вся масса распределена в толще объёма корочки поверхности, от совсем фантастических пары миллиметров до чуть более реалистичного километра, а внутри - пустая полость?
297 492289
>>92288

>Если у нас будет планета-мяч, где вся масса распределена в толще объёма корочки поверхности, от совсем фантастических пары миллиметров до чуть более реалистичного километра, а внутри - пустая полость?


Внутри сила тяжести будет нулевая. Другое дело, что это нереально, потому что твоя корочка просто схлопнется.
298 492303
Почему у полой земли внутри должна быть невесомость? Ведь внутренняя стенка, которая рядом и ближе, не должна уравновешиваться точно такой же стенкой но в 12 000 км от тебя, ввиду квадрата расстояния.
299 492324
>>92289

> Внутри сила тяжести будет нулевая.


А снаружи? На поверхности?
Вот стоишь ты на поверхности такой пустой искусственной планеты и кидаешь на пол яблкок, оно будет падать без ускорения, грубо говоря, в слоу-мо?
BDaM1CCMAAxrrN.jpg63 Кб, 599x736
300 492329
>>92324

>яблкок


Эпплкок?

>> Внутри сила тяжести будет нулевая.


Да как вы это считали? В центре только нулевая. При вращении будет обычный шар О'Нила.

>А снаружи? На поверхности?


Считаешь массу, если оболочка одной плотности и толщины - центр масс будет в центре сферы, можно считать обычную силу притяжения. Алсо если эта йоба вращается - можно сходу с экватора улететь. Если масса очень маленькая, не многабольше твоей, можно ногами раскрутить, будет беговая дорожка. Только надо как-то удерживаться на поверхности, а то от первого шага улетишь.
301 492330
>>92303

Потому что интеграл считай, вот почему.
302 492332
Вот щас всякие программы по освоению луны готовятся, а зачем? Неужели за 17 Аполлонов там не всё изучили?
14792942131690.webm3,6 Мб, webm,
416x320, 1:48
303 492333
>>92329

>Алсо если эта йоба вращается - можно сходу с экватора улететь


У нас на Земле это постоянно.
304 492334
>>92332
А чё они там изучали?
Сидели-пердели в шаттле, из которого выходили пробздеться и потоптаться по поверхности. Камушки с поверхности невадской пустыни пособирали, а на глубину хотя бы пары метров ясен хуй никто не сувался и ничего не исследовал. А вдруг там нефтьсеребро, платина, ирридий?
305 492336
>>92334

>а на глубину хотя бы пары метров ясен хуй никто не сувал


А зачем? Что это знание даст лично тебе?
306 492337
>>92334

>А вдруг там нефтьсеребро, платина, ирридий?


Даже есть так, то что? Возить от туда на Землю сырье экономически нецелесообразно.
307 492338
>>92303
Грубо говоря, потому что противоположной стенки намного больше чем ближней. Ровно во столько раз, во сколько ближняя сильнее тянет. Строго доказывается интегралом, "на пальцах" - нарисуй два конуса с общей вершиной и прикинь площади основания в зависимости от высоты.
308 492341
>>92324
Снаружи будет ускорение ровно такое же, как если бы твоя планета была всюду однородной. Другое дело, что оно будет слабым. Прикинь, во сколько раз меньше Земли масса у ее километровой "корочки".

Вообще, избежать "наружного" тяготения невозможно, если у твоей планеты есть хоть какая-то масса. Внутри - да, потому что, грубо говоря, планета находится "по разные стороны от тебя" и сила тяжести туда и сюда банально уравновешивается (при определенной форме планеты). А снаружи вся планета по определению будет "с одной стороны от тебя", и именно туда тебя и будет притягивать.
309 492349
>>90880
Во-первых Бамп

И затем вполне геологичный вопрос. Вот есть магнитное поле земли. Можно ли проследить его суточное изменение в разных точках планеты исключая солнечный ветер?
310 492382
>>92329

>Да как вы это считали? В центре только нулевая. При вращении будет обычный шар О'Нила.


Внутри сферы гравитация везде 0, считай интеграл.
ad1e8cf685549d47828525b3abef920782.png15 Кб, 654x296
311 492391
>>92382
Такой? Спасибо, я поход упырок тупой. Надо повторять все, смотрю на интеграл, как на фото из детского сада, вроде было такое, но что с этим всем делать - не помню.
312 492392
>>92391

>что с этим всем делать


Очевидно же, скормить вольфраму.
313 492393
>>92392
И типа признать, что вольфрам умный, а я так и останусь тупым упырком? Я лучше учебничек скачаю и обмажусь по новой, через силу, через боль.
314 492394
>>92393
Да нахуй надо, все равно же через месяц забудешь, если не будешь постоянно этим пользоваться.
315 492396
>>92394
Не, ну я маленько помню что с интеграалями делать, прост матан был больной темой. И сходу не решу. Из технических вроде единственная позорненькая троечка в выкидыше-вкладыше.
316 492404
317 492414
>>92404
Что-то похоже на хуету, даже если результат и правильный. Там еще делить на косинус укла падения надо, если не ошибаюсь. Правда, эти косинусы всегда равны.
318 492415
>>92414

>Там еще делить на косинус укла падения надо


Какого падения, там берётся бесконечно малый кусочек поверхности, он всегда перпендикулярен лучу.
319 492416
>>92415
У тебя луч - хорда, не проходящая через центр в общем случае. Она нихуя не перпендикулярна поверхности сферы.
320 492417
>>92415
>>92416
Два дебила - это сила.
321 492424
>>92404

>доказывается


Ну немного сильно сказано. Там, поправь, если не прав, доказывается, что провели произвольную прямую и для двух противоположных крошечных кусочков сумма сил равна нулю, а потом так - "с точки зрения банальной эрудиции, какие бы два других таких не взяли - будет так же". Это к выводу интеграла какбэ подвели, а потом "ну вы все равное дебычи, давайте без решения прикинем, один хуй считать сложна!". Разве нет?
322 492492
>>92424
Угомонись, олигофрен.
323 492527
Как образуются двойные астероиды? У них же гравитация такая милипездрическая, что говно может покинуть их сферу притяжения просто от пердежа. Но они каким-то образом образуются и еще тройные даже астероиды как-то есть.
324 492528
>>92527
расклеиваются
325 492548
>>92527
Так они пердеть не умеют просто. сириусли
Просто за дохуя сотен миллионов лет случается такое, что два астера прилетают в одну точку, причем зачастую не совсем случайно, а в каких-нибудь низкопотенциальных точках.
326 492563
Что такое облако Оорта??
В википедии очень неясные определения
15280285314980.jpg147 Кб, 858x932
Денис 327 492565
Почему-бы не сделать международную автоматическую обсерваторию на Луне с гигантским телескопом - значительно более крупным чем Хаббл, размерами на уровне крупнейших земных. Ведь на Луне отсутствует атмосфера, по сути на её поверхности открытый космос что очень благоприятствует наблюдениям
Даже такие богатые страны как США или КНР в одиночку не потянут такой проект, но вместе вполне, можно ещё подключить ЕС, Индию и Россию
328 492566
>>92565
Затем что в космосе делать обсерваторию проще, чем на луне.
329 492567
>>92565

>Почему-бы не сделать международную автоматическую обсерваторию на Луне


А почему бы ее не сделать на земной орбите, ближе к Земле, а значит проще и дешевле. В чем профит нахождения именно на Луне?
330 492570
>>92567
надёжнее - Луна всё-таки планета. При малейшей аварии этот орбитальный надземный телескоп может полностью утерян.
rogozin moon.png399 Кб, 633x781
331 492571
>>92570

>Луна всё-таки планета



Рогозин, ты?

>При малейшей аварии этот орбитальный надземный телескоп может полностью утерян.



А на Луне аварий быть не может? Туда слишком дорого отправлять нагрузку, и в космосе менять точку наблюдения намного проще.
332 492572
>>92570

>При малейшей аварии этот орбитальный надземный телескоп может полностью утерян.


Можно подумать на луне он не будет утерян при малейшей аварии.
Если же телескоп такой ценный что оправдывает спасательную миссию то на земной орбите миссия выйдет гораздо дешевле.
Максимум что на орбите его может закрутить так что нельзя будет стыковатся и придется как то его тормозить. Но для НОО этим процессом можно нормально управлять удаленно, пинг позволяет.
333 492573
>>92571

>в космосе менять точку наблюдения намного проще


Можно Луну поворачивать.
334 492574
Поясните, что выгоднее, запустить 2 ракеты с грузоподъемностью 10 тонн или 1 ракету на 20 тонн? Или однохуйственно?
AndrewFeustelperformsworkontheHubbleSpaceTelescope.jpg822 Кб, 3072x2034
335 492577
>>92570

>При малейшей аварии этот орбитальный надземный телескоп может полностью утерян


К тому же Хабблу как раз летали и обслуживали его. Цена же ремонта на Луне же на порядки больше, людей на это дело посылать точно никто не будет. Еще вопрос, что будет дешевле, запустить новый телескоп на орбиту вместо потерянного или отправить команду ремонтников на Луну.
336 492578
>>92572
на орбите такой объект будет хрупким и уязвимым. От малейшей аварии может сойти с орбиты
337 492580
>>92578
Ага бля, в занос уйдет - и в кювет.
15600083046560.jpg728 Кб, 900x900
338 492581
>>92577
на Луне можно будет построить атомную станцию. Радиотелескопы
Склады, космодром - своеобразный перевалочный пункт и плацдарм для полётов на Марс и прочие планеты, места для проживания космонавтов, астронавтов и прочих тайконавтов
339 492583
>>92580
может как астероид свалиться на пендосов или хохлов
340 492584
>>92581
В любом случае стоимость, время и сложность доставки грузов и людей до орбиты будет гораздо меньше, чем до Луны. Если задача иметь космический телескоп, то выгоднее иметь его на орбите, если Луна не может дать каких-либо существенных бонусов.
341 492594
Сап, ананасы,
Почему схема с несколькими пусками и сбором корабля на орбите считается менее удачной в сравнении с разовым стартом на сверхтяже? Какие там небесные камни?
Screenshot2019-07-07-09-45-42.png312 Кб, 480x854
342 492597
>>91219
Пираты одобряют
>>92578
А что его на Луне защитит?
343 492598
>>92581
А чего уж не на Титане сразу?
344 492634
>>92594
стыковаться сложно и опасно
345 492638
>>92391
Можешь для тренировки внутри не сферической, а кубической оболочки силу гравитации посчитать.
346 492650
>>92548
Чет слабо верю, что орбита и скорость может совпасть случайно. Это как если мусор на орбите случайно рандеву и стыковку с МКС сделает.
347 492651
>>92382
Интеграл тут не нужен, можно доказать проще на пальцах. Если от любой точки внутри сферы построить бесконечно малый объемный угол (кажется так называется, не помню уже) в противоположных направлениях, площадь и, соответственно, масса участков поверхности, в которые он упирается, будет пропорциональна квадрату расстояния до них, так что создаваемое ими гравитационное притяжение уравновесится. По сути то же интегрирование, но таким образом, что ноль получается сразу.
EarthinultravioletfromtheMoon(S72-40821).jpg436 Кб, 1719x1362
348 492654
>>92598
на Титане далеко, там туман. Говорят с Титана не видно даже Сатурна.
А на Луне можно установить целый набор телескопов - в северном и южном полушарии Луны, на полюсах, на видимой и обратной стороне. Целые книги по лунной астрономии можно писать. Кто-нибудь проводил расчеты кстати, когда там транзиты Венеры и Меркурия будут видны с поверхности Луны, они же не будут совпадать с транзитами с Земли? Не проводили, потому что не думал что там может появиться наблюдатель, а он там может и должен появиться.

С Луны можно и за Землей наблюдать во всех спектрах. Аполлон 16 например сняли вот эти кадры Земли с поверхности Луны в ультрафиолете. Видна дневная и ночная сторона, как тепло распределяется. Это может быть полезно для построения моделей поиска экзопланет, нам нужно знать как наша планета выглядит с расстояния, чтобы потом искать такие же сигнатуры у других звезд. Можно лунные и солнечные затмения наблюдать с Луны, это круто наверное видеть как Солнце заходит за Землю, оно длится очень долго и в это время просвечивает атмосферу Земли, так что также можно изучать по разному. Может мы даже выхлопы заводов там таким образом увидим?
349 492655
>>92651
При подобных рассуждениях очень легко ошибиться.

Такими рассуждениями, как у тебя, анализ и начинался в истории математики. И даже многого правильного тогда обнаружили.Но нередко бывало и так: после подобных рассуждениях "на пальцах" один математик получает один ответ на задачу, а другой - совсем другой, противоречащий первому. Когда всем надоело спорить, кто же прав на пальцах, - поняли, что надо работу с бесконечно малыми величинами на какую-то строгую основу поставить.
350 492656
>>92597

>А что его на Луне защитит?


На Луне можно построить бункер изолированный от тепла и холода, от микрометеоритов, прикрытый реголитом.
351 492665
>>92655
В формализованном виде эти рассуждения есть обычная замена переменной - стандартный прием при взятии интегралов. В качестве переменной берем силу в данном направлении, получаем ноль сразу.
352 492668
>>92665
Смотри, например, в чём одна из проблем рассуждений на пальцах.

> Если от любой точки внутри сферы построить бесконечно малый объемный угол (кажется так называется, не помню уже) в противоположных направлениях, площадь и, соответственно, масса участков поверхности, в которые он упирается, будет пропорциональна квадрату расстояния до них



А ведь эти участки не будут строго подобными геометрическими фигурами с таким-то коэффициентом подобия.
Они будут не только разного размера, но и немного разной формы. Пусть один участок, например, маленький-маленький "кружок" - "сферический сегмент", если правильно говорить. Тогда другой - не обязательно настоящий сферический сегмент, он может быть слегка "кривоватым" сферическим сегментом - тем кривоватей, чем больше радиус нашего исходного маленького "кружка". И соотношение между площадями будет не такое простое, как у обычных подобных фигур.
Но мы этой кривоватостью пренебрежём, так? У нас сфера огромная, а маленькие участочки - ну очень маленькие, бесконечно маленькие, поэтому и кривоватость - бесконечно маленькая, так?

Тогда смотри вот ещё одно рассуждение с таким же пренебрежением.

Я хочу посчитать длину дуги параболы Y = X^2 при X от 0 до 0,1. Разбиваю этот отрезок [0; 0,1] на кучу ну очень маленьких, бесконечно маленьких участочков. Раз они такие маленькие, то Y на каждом из них практически не успевает измениться. Следовательно, можно считать, что на каждом график функции идёт просто горизонтальным отрезком. Ну, да, не совсем ровным отрезком, слегка кривовато - но кривоватость же бесконечно маленькая, пренебрежём ей!
Значит, длина графика - сумма длин этих маленьких горизонтальных отрезочков. Которая, очевидно, и равна 0,1.

В чём разница? Почему там можно было пренебрегать, а там нельзя?

Хотя задним числом-то, зная правильные ответы, можно, конечно, так же на пальцах высосать из пальца причину, почему там - можно, а там - нельзя. А вот что ты будешь делать, если правильного ответа не знаешь и должен его найти?

Ну-ка, скажи мне: внутри куба тоже в каждой точке невесомость? А внутри пирамиды? А внутри клинокороны? А внутри тора? И насколько ты в ответах уверен - не считая, а прикинув на пальцах?
352 492668
>>92665
Смотри, например, в чём одна из проблем рассуждений на пальцах.

> Если от любой точки внутри сферы построить бесконечно малый объемный угол (кажется так называется, не помню уже) в противоположных направлениях, площадь и, соответственно, масса участков поверхности, в которые он упирается, будет пропорциональна квадрату расстояния до них



А ведь эти участки не будут строго подобными геометрическими фигурами с таким-то коэффициентом подобия.
Они будут не только разного размера, но и немного разной формы. Пусть один участок, например, маленький-маленький "кружок" - "сферический сегмент", если правильно говорить. Тогда другой - не обязательно настоящий сферический сегмент, он может быть слегка "кривоватым" сферическим сегментом - тем кривоватей, чем больше радиус нашего исходного маленького "кружка". И соотношение между площадями будет не такое простое, как у обычных подобных фигур.
Но мы этой кривоватостью пренебрежём, так? У нас сфера огромная, а маленькие участочки - ну очень маленькие, бесконечно маленькие, поэтому и кривоватость - бесконечно маленькая, так?

Тогда смотри вот ещё одно рассуждение с таким же пренебрежением.

Я хочу посчитать длину дуги параболы Y = X^2 при X от 0 до 0,1. Разбиваю этот отрезок [0; 0,1] на кучу ну очень маленьких, бесконечно маленьких участочков. Раз они такие маленькие, то Y на каждом из них практически не успевает измениться. Следовательно, можно считать, что на каждом график функции идёт просто горизонтальным отрезком. Ну, да, не совсем ровным отрезком, слегка кривовато - но кривоватость же бесконечно маленькая, пренебрежём ей!
Значит, длина графика - сумма длин этих маленьких горизонтальных отрезочков. Которая, очевидно, и равна 0,1.

В чём разница? Почему там можно было пренебрегать, а там нельзя?

Хотя задним числом-то, зная правильные ответы, можно, конечно, так же на пальцах высосать из пальца причину, почему там - можно, а там - нельзя. А вот что ты будешь делать, если правильного ответа не знаешь и должен его найти?

Ну-ка, скажи мне: внутри куба тоже в каждой точке невесомость? А внутри пирамиды? А внутри клинокороны? А внутри тора? И насколько ты в ответах уверен - не считая, а прикинув на пальцах?
353 492670
>>92634
Но ведь стыковки на орбите происходят постоянно. Я думал, на примере мкс там все давно отработано и рутина, не?
354 492671
>>92391

> Такой?


Нет, не такой. Ты, насколько понимаю, считаешь для "толстой сферы": шара радиуса R2, из которого вынули шар радиуса R1 (где R1 и R2 близки друг к другу, но всё-таки разные, R1 > R2). А тут нуля и не получится.

Внутри "толстой сферы" ускорение свободного падения будет равно нулю только в самом центре, а при удалении от центра - понемногу возрастать, тем ощутимее, чем сфера "толще".

Ньютон доказал, что везде в точности ноль, для идеально тонкой сферы.
355 492672
>>92668
В общем случае так не будет, т.к. играет роль угол, под которой участок поверхности расположен (в твоем примере с параболой то же). Для сферы несложно доказать, что для противопоставленных участков эти углы равны, у других фигур так не будет. Искажения формы участков не влияют на результат, они другого порядка малости (емнип, даже какая-то теорема есть про это). Это даже не матан, это школьная программа.
356 492673
>>92672
Понятие порядка малости - не школьная программа, это именно мат. анализ.
6050692-5182240-5168925-neq2dzdcrlqgty2a.jpg134 Кб, 1200x633
357 492674
>>92668

>А внутри тора?

358 492675
>>92671
Принцип суперпозиции же. Разделяешь толстую сферу на бесконечно тонкие, интегрируешь по ним R1->R2, в сумме получаешь тот же ноль.
359 492677
>>92675

> Принцип суперпозиции же.


Употребление умных слов в сочетании с глупыми утверждениями смотрится очень плохо.

Угу, особенно здорово - когда R1 = 0. То есть обычный заполненный шар. Закопался такой в пещеру внутри Земли - и плаваешь там в невесомости! Принцип суперпозиции же! А от нуля тоже можно интегрировать!

А, что, нельзя от нуля, R1 у нас должен быть положительным? Ну ладно! Пусть R1 = 1 миллиметр. Злые инопланетяне поместили в центр ядра Земли пустой шарик радиуса 1 мм. Бац! Бум! Трах! Отключилась гравитация внутри всей Земли ниже поверхности!
360 492678
>>92677

>особенно здорово - когда R1 = 0


В самом центре Земли невесомость, в чем проблемы?
361 492680
>>92677
Тащемта доказательство для точек с r<r1 т.е. когда ты под землей, на тебя не действует та часть планеты, которая дальше от центра.
362 492683
А, да, ерунду написал.
363 492690
>>92673
а че, начала матана в школке уже не проходят?
364 492696
365 492698
>>92670
На МКС отработана стыковка МКС с конкретными грузовиками. Сама сборка МКС была ебучим геморроем. В космосе вообще всё сложно, даже просто гайку закрутить.
366 492725
>>89400 (OP)
На вебм магнитное поле сраной земляшки я так понимаю? Почему его так пидорасит?
367 492726
>>92725

этого никто не знает
368 492727
>>92725
нет, ты довн, не магнитное поле, а поле заряженных частиц, хотя небольшая корреляция всё же есть
колебания - это колебания доминирования солнцем земли
369 492728
>>92727

>а поле заряженных частиц


Которые задерживаются магнитным полем сраной замляшки, так?
Соответственно благодаря этим частицам, мы видим контуры магнитного поля.

>>92727

>ты довн


Это тред тупых вопросов если что. Довн здесь только ты. ^_^
рогозин 4.png445 Кб, 1200x628
370 492729
>>92573
проиграл
371 492733
>>92728

>Которые задерживаются магнитным полем сраной замляшки, так?


>Соответственно благодаря этим частицам, мы видим контуры магнитного поля.



Не, ты не так понял. Магнитное поле Земли так сильно не меняется, на таких масштабах его вообще можно постоянным считать. Отток частиц из поясов тоже примерно постоянен.

Единственный изменяющийся параметр это приток заряженных частиц со стороны Солнца, высрало оно большую вспышку, например, долетело до Земли больше частиц, ну они на несколько месяцев в земных радиационных поясах и застряли. Не высрало — ну и застревать, соответственно, нечему.

На вебмке показана именно интенсивность потоков частиц, а не какая-нибудь там напряженность поля.
243Ida-August1993(16366655925).jpg93 Кб, 628x380
372 492779
1)есть какие-то прикидки, у какого процента астероидов есть такие спутники?
2)можно так примерно почувствовать, когда будут первые подтверждённые экзолуны? Следующее поколение телескопов вроде JSWT сможет их засечь как-нибудь? Какие для этого методы есть, ловить транзиты планеты?
373 492780
>>90817

>Проще лазером прожечь дырку в скафандре.


Учитывая, что лазырь 98% процентов потребляемой энергии перегоняет в тепло, лазер способный прожечь дырку в скафандре будет иметь радиатор размером в небольшой парус.
71ohVh5TSHL.SX425.jpg36 Кб, 425x425
374 492781
>>90834
Обо всём подумали до нас.
375 492812
какая скорость гравитации?
На википедии нет однозначного ответа и есть предположение что мгновенна во Вселенной
376 492822
>>92812

>какая скорость гравитации?


c

>есть предположение что мгновенна во Вселенной


Хуёвое предположение.
377 492838
>>92812
Заебали. В шапку надо добавить про результаты наблюдений LIGO слияния двух нейтронок, когда гравволны пришли практически одновременно с гамма-всплеском.
http://ru.wikipedia.org/wiki/Слияние_нейтронных_звёзд
378 492913
>>92838

>практически одновременно с гамма-всплеском.


Раньше или позже?
379 492915
>>92913
нельзя точно сказать, так как сперва детектор гравиволн работает во все стороны, а источник гамма-лучей надо находить потом, по результатам первой детекции, минут 10 задержка в лучшем случае между обнаружением гравиволн и определением источника и его излучения
380 492959
Почему у ракет кпд около 1%?

Почему ракеты такое говно?
4b241a50b13c495ba1c93bd16a458f0e.jpg476 Кб, 1033x1462
381 492960
Часто вижу яркую жёлтую немигающую точку на южной стороне неба довольно низко над горизонтом. Светит всю ночь, перемещаясь с юго-запада на юго-восток. Хрен что разглядел в подзорную трубу с 10-кратным увеличением, но кажется, будто это Сатурн - был какой-то намёк на кольца. Я прав?
382 492965
>>92959
более высокие варианты кпд имеют очень хуевое соотношение тяги к весу и не способы подняться в космос
383 492971
>>92965
А как же двигатель шаубергера?
384 492972
>>92971
иди нахуй
385 492982
>>92960
Присоединяюсь к вопросу. Я грешу на Марс.
Мимо-ньюфаг
386 492994
>>92972
Что иди нахуй долбоеб?
Почему двигатель шаубергереа не юзают в космонавтики и пускают хим питарды с кпд 1 процент 60 лет уже?
Нефтянники ротшильды запретели космос тупым гоям?
388 493013
>>93002
Трудно что-либо разобрать. Как я понял, это действительно скорее всего Сатурн, либо Юпитер. На карте они отмечены рядом, но на небе я видел только одну планету.
389 493014
>>92959

>Почему у ракет кпд около 1%?


У ракет КПД не 1%, а в определённых участках полёта может превышать 100% (ракета сосёт дополнительную энергию из гравитационного манёвра).

>>92971

>А как же двигатель шаубергера?


Лежит в тайной комнате сразу за философским камнем, без гербалайфа не заводится. Вход - три мавро.

Вообще, вечный двигатель 2 рода запрещён не то чтоб совсем фундаментально, но здесь придётся выходить сильно за пределы молекулярной материи и привычного нам макромира. На трубках с водой ты его не запилишь.
sage 390 493021
>>92915
Шо ты несешь, маня. Статью прочесть не судьба?
15621347429610.jpg138 Кб, 1280x719
391 493049
В начале года в сми звучали вскукареки, что какой-то брит симуляцией смог связать темную энергию и темную материю. Что, мол, межгалактическое пространство заполнено какой-то "жидкостью", которая одновременно давит на галактики снаружи, не давая им разлететься (т. материя), и в тоже время увеличивает расстояние между самими галактиками, как бы раздвигая их (т. энергия). На сколько это серьезно, и что с этой гипотезой сейчас?
392 493122
>>93013
А ты указал точную дату, время и месторасположение? А направление? А высота над горизонтом?
393 493123
>>93049
Но ведь ТМ аккуратно кучкуется вокруг и внутри галактик, в межгалактической среде ТМ разных галактик редко пересекаются.
394 493171
>>93122
Да, но всё равно настоящая картина неба сильно отличается от той, что на этих картах. Видимо виновата засветка, или моё плохое зрение.
395 493210
Почему люди не могут построить станцию бублик с гравитацией? Мсполтзовать как перевалочный пункт/лабораторию?
396 493211
>>93014
Как может быть более 100, если 95 процентов ракеты это ее горючка? Ракета по сути саму себя возит
397 493221
>>93211
КПД не имеет никакого отношения к массе. КПД это про джоули и ватты, блядь.
398 493224
>>93014
тупой вопрос - при грав манёвре модуль скорости меняется или нет?
399 493230
>>93224
Скорости относительно чего? Относительно Солнца меняется, потому что относительно планеты аппарат меняет вектор, и за счёт этого пиздит у планеты часть кинетической энергии.
400 493248
Почему у Сатурн-5 диамерт обтекателя такое дно - 6 метров? Даже Нью глинна обещает 7 метров, хотя в другом классе, не говоря уж об воображаемом китайском сверхтяже с 10 метрами.
401 493249
>>92779
1) Сложно сказать - когда к Иде послали зонд не знали что у неё спутник есть, так и сейчас могут быть много астероидов со спутниками, но мы просто не в курсе. Мы можем какие-то статистики по большим двойным астероидам рисовать, а не по с астероидам с маленькими спутниками.
2) Примерно почувствовать можно так - в ближайшие годы. Был уже один кандидат в экзолуну, но не подтвержден на 100%. транзиты, да
402 493253
>>93230
относительно планеты, об которую маневрирует
403 493282
>>93253
Относительно планеты не меняется.

Можешь грубо представить, что вместо планеты в космосе движется Г-образная труба. В одну дырку аппарат влетает, в другую без трения вылетает. Но направлены эти дырки относительно вектора скорости трубы по-разному, и за счёт этого мы имеем бесплатную дельту.
404 493291
Почему ещё не один ровер не послали на полярные шапки Марса? Там же всё самое интересное.
22i103-104-105-109FROST.jpg424 Кб, 2311x678
405 493293
>>93291
Викинг 2 ландер садился в высоких широтах. Изморозь видели на камнях. Но смотри какие камни там, пиздос, как по такому ездить на ровере?
407 493295
>>93294
Хотели один такой послать - смотри видео >>493250
408 493437
>>93210
1. Дораха
2. Космические лаборатории для микрогравитации и городят, обычного "жэ" и на Земле хватает
3. Сложно стыковаться.
409 493441
>>93210

>Почему люди не могут построить станцию бублик с гравитацией? Мсполтзовать как перевалочный пункт/лабораторию?


Тупо дорого, потому что для приемлемых сил кориолиса получается такая огромная дура, что МКС сосёт просто. Даже для станции с противовесом дура огромная. Это уже не перевалочный пункт и не лаборатория, а орбитальный городок.

Ща надувные модули отработают, многоразовые сверхтяжи по 100$ за кг, может катапульту какую-нибудь придумают - и тогда уже будут играться в мегаструктуры.
410 493451
>>93441
Анон, ты уточняй для ньюфагов, что твое "Ща отработают" это по астрономическим меркам, а не по профанским, то есть через лет 50.
411 493455
>>93451
Это не ко мне, это к политикам. Дайте мне фюрерскую кепку - и всё это можно будет запилить лет за 10, но не без массовых расстрелов, разумеется.
634r5.jpg20 Кб, 512x384
412 493458
>>89400 (OP)
Что это за два тела видны в марсианском небе? Фантастический фейк или ирл онм так и выглядят (и так рядом друг с другом) с планеты?
413 493462
>>93458
Нет. Деймос вообще выглядит как звёздочка. Фобос - камень, но не настолько большой.
Впрочем, обожаю Марс в этом фильме. Ничего общего с реальным по освещённости, цвету и спутникам, но блин, насколько же круче и атмосфернее настоящего Марса выглядит.
414 493464
>>92334

> а на глубину хотя бы пары метров ясен хуй никто не сувался и ничего не исследовал.


Они около какого-то вроде типа метеоритного кратера же приземлялись, там поди срез пород был куда покруче, чем 2 метра
image.png2,6 Мб, 1304x1630
415 493465
>>93458
Фобос и Деймос
ИРЛ намного меньше, ближний Фобос вот так выглядит, Деймос дальше и меньше, его вообще наверное хуй отличишь от других точек на небе
phobos-deimos-moon-comparison-e1524075097586.jpg14 Кб, 800x600
416 493468
>>93458
Вот Фобос и Деймос ИРЛ, по сравнению с Луной.
417 493485
>>93462
Что за фильм?
418 493502
>>93462

>Впрочем, обожаю Марс в этом фильме. Ничего общего с реальным по освещённости, цвету и спутникам, но блин, насколько же круче и атмосфернее настоящего Марса выглядит.


на горе Олимп всё гораздо интереснее
Видимо места посадки марсоходов довольно непримечательные
419 493504
>>93502
да они специально скучные места посадки выбирают, потому что они безопасные. Плюс, камеры марсохода низко расположены, медленный он. Вот послали бы рой вертолётов на Марс, самолёт, дирижабль, аэростат, сняли бы нормальней интересных мест.
420 493518
>>93504

>Вот послали бы рой вертолётов на Марс, самолёт, дирижабль, аэростат


...и отсосали бы хуйцов в атмосфере плотностью в доли процента от земной. Летать там, конечно, можно, но низэнько-низэнько и очень недалеко. Джетпак там, пожалуй, экономичнее любой аэродинамической и аэростатической машины.
421 493519
>>93518
переделать можно под марсианские условия, там гравитация ниже
422 493533
>>93519
Гравитация ниже в три раза, а плотность в 100+ раз. Лучшие конструкции из ёба-полимеров там будут летать хуже фанерных этажерок на заре авиации.
423 493536
>>93533
ой, такие как ты сказали бы что дрон-вертолёт на Марсе невозможен, вот только НАСА уже сделала его и испытала, так что завали ебало. Про аэростаты на Марсе писали статьи еще в совке, а самолёту нужно иметь лишь побольше крыло.
424 493555
>>93518
Скоро нанодроны начнут ебашить, так что не все потеряно
425 493571
У меня стоит вопрос, если сжигать метан и кислород под давлением 200 атмосфер с целью отопления, будет ли экстра кпд?

То есть раз в сутки подогревать тепловой аккумулятор с помощью ЖРД

во сколько раз выигрыш?
426 493584
>>92334
Кек.
Кернов нахуярили прилично. Автоматы так до сих пор не могут.
Да, ручками.
Депутат Серова из космоса смотрела на гибель людей в Донбас[...].webm5 Мб, webm,
640x360, 1:02
427 493598
Пиздит ли Серова или нет? Можно с МКС взрывы увидеть? А траектории?
428 493650
>>93282
спасибо, братишка
429 493666
>>93598
Хочешь двушечку за неуважение к власти?
430 493682
>>93666
Здесь не /po/раша, я бы хотел ответы на мои вопросы.
431 493686
>>93682
Ну так не тупи и погугли "фото земли с мкс" и прикинь что там можно увидеть на фотографиях
432 493687
>>93682
Да нухуя ты там, естественно, не увидишь.
433 493700
>>93686
>>93687
Написал донос куда следует.
434 493707
>>93571
Выигрыш по сравнению с чем?
jim-carrey-dr-robotnik-sonic-the-hedgehog.jpg92 Кб, 1213x667
435 493718
Вояжер один и 2. В вики ссылаясь на сайт наса, который никуда не ссылается, написано, что они пролетят мимо AC+79 3888 в 1.6 св. г. через 40к лет, первый, а так же в то же время второй мимо Росс 248. 1.7 св.г. и через 296к лет мимо Сириуса в 4.3 св.г.

В связи с этим 4 вопроса.
1. Это с учётом движения звёзд друг относитлеьно друга?
2.Есть ли более нагляднный график или более точные данные, дабы самому построить данный график, а не сухие "1.7 св. г."?
3. 4.3 св. г. это примерное расстояние между звёздами. Значит ли это что любых других звёзд рядом не будет или сириус взяли для красного словца и будет куча звёзд?
4. Учтено ли отклонение аппаратов при сближении со звёздами? Быть может это отклонение пренебрежимо мало?
436 493719
Видел видео про то, что психея может быть ядром бывшей протопланеты. Значит ли это, что сказка про Фаэтон не совсем необоснованна?
437 493735
>>93718

> есть ли


короч берёшь у RECONS'ов данные по собственным движениям звёздочек, при нужде добавляешь из крайней гайи
и считаешь шо хочешь
438 493737
>>93719
сказка про Фаэтон говорит о разрушении уже сформировавшейся планеты, а психея вряд ли была когда-то планетой, она не смогла набрать нужную массу, как шкет, который начал рано курить и остановился в росте.
439 493740
Как объясняется минимум Маундера?

Почему у Солнца бывают периоды когда на нем нет пятен? Почему в такие периоды температура на Земле снижается?
440 493749
>>93740

> на Земле


температурка зависит от температуры на солнышке, а когда на нём пятна - оно теплее
441 493831
>>93749
Так ведь наоборот же, пятна это холодные области, значит с пятнами солнце холоднее.
442 493853
Сап космач.

Гравитация в космосе это хорошо, однако пилить большой бублик диаметром более 500 метров как то трудно, нет средств доставки да и капитализм порешал, никакого космоса и полетам не будет, ибо ротшильдам нет выгоды.

Однако, зачем пилить бублик, если можно все упростить.
Смотри анон - центр станции будет неподвижный модуль, состоящий из дока и лабораторий для невесомости. А вокруг всей этой ебалы крутитья два модуля, соединенные коридором длинной 500 метров. Один модуль будет жилым/медицинским, а второй для склада и прочего (лаборатории, оранжерея и т.п.

Иными словами, это просто упрощенный бублик, только вместо него два модуля соединенные длинным коридором которые вращаются вокруг причального модуля

Как идея? Вроде все просто, нет никаких подводных камней.

Только как это построить все. Можно ли использовать какой нибудь космический бетон/полимер, чтобы лить прямо в космосе?
002.jpg102 Кб, 660x371
443 493860
>>93853
Берёшь, блядь, ферму как у подъёмного крана, метров на сто, а лучше на двести. Посередине пускаешь лестницу в брезентовой трубе, чтобы человечки по ней ползали. По бокам два модуля+двигатели раскрутки, посередине причальный.
Этой идее сто лет в обед. Пока оно нахуй никому не упало, потому что на МКС летают ради невесомости. Когда понадобится космический хаб для обслуживания колонии и долговременного пребывания кучи человеков - тогда сделают.
444 493862
>>93860
Но зачем тебе долговременно пребывание на такой миллипиздрической хуйне? По сути долговременное пребывание необходимо только если ты собираешься селить миллионы людей, чтобы они там дальше ебались и воспроизводились. Тогда в любом случае тебе только исполинское колесо остаётся делать.
445 493867
>>93860
А как в космосе создавать квадратные строения? Ведь размер модуля ограничин габаритами грузового отсека ракеты. И все блядь круглое.

Нет еще какого нибудь космического бетона или кирича, чтобы прямо на орбите отливать собирать станцию
Да, чому бы из бетона не лить? Ремонтопригодность супер, прочный.
446 493874
>>93867
Вся эта ебала негерметичная, то есть тебе в любом случае нужен будет внутренний металлический корпус, причем сплошной. А тогда нахуя тебе вообще бетон? Нагрузки там пока не те, чтобы выцеживать охуительную прочность (армированный бетон лучше металла практически только тем, что дает сравнимую прочность при меньшей себестоимости).
447 493880
Что там с тем квестом по мотивам интерстоляра, где надо 20 лет на орбите заниматься исследованиями?
448 493891
>>93874
Прсто бетоном проще любые формы и размеры пилить. Даже модуль 150 на 50 метров ракеиой не вывезти. А так проще было бы пилить прямо в космосе.
Почему бетон негерметичный? Ну можно не из чистого бетона, а возможно усовершенствовать его. Или полимерной пленкой покрыть внутри.

Плюсы железобетона такие:

1. Можно пилить строения любого размера и формы
2. Легко чинить. Любую пробоигу можно замазать.
3. Легко достраивать новые модули.
449 493929
>>93891
А опалубка? Полимеры разрушаются радиацией.
Но у бетона так же есть один серьёзный недостаток - он крошится, в то время как металл гнётся. И микрометеориты с космическим мусором за такое только спасибо скажут. И не говоря о том, что ты не сделаешь бетон 5мм толщиной, тебе надо пилить сотни тонн, в то время как с модульным металлом, ты управишься в разы меньше.
В итоге мы получим вдвое худший результат при таких же затратах. Орбитальная сборка имеет смысл лишь когда у нас на орбите уже есть стройматериалы в виде метеоритной руды или лунной. А если завозить разобранные модули, то по весу они будут те же, а сборка будет более худшая из за херовости оборудования в сравнении с земным, + незабываем о нештатных ситуациях, вроде той что случилась с паукоро-марсоходом. Если бы это случилось на орбите пришлось бы отправлять ещё одного, а так ты отправляешь другого не тратя на 2 ракеты. + многие производства требуют расходников, и их тоже придётся достовлять на орбиту.

Модульный комплекс типа мкс это пока единственная отработанная технология на ближайшие сотни лет. И единственный минус это узкие двери (1.5 метра) и комната 4 на 10 метров. Если для тебя это маленькая комната. то я даже не знаю.
450 493977
>>93929
Ну хорошо, согласен с тобой

Слушайте. Тут физики есть?
Я читал про гравитационные волны. Написано, что их порождает либо массивное иело либо тело с асиимитричным ускорением. Что такое ассимитричное ускорение?
sage 451 493978
>>93891

>Прсто бетоном проще любые формы и размеры пилить. Даже модуль 150 на 50 метров ракеиой не вывезти. А так проще было бы пилить прямо в космосе.



Бетон лить в космосе? Тащить цемент, песок, воду, бетономешалку м прямо в открытом космосе мешать? Ты подкуренный что ли?
452 493979
>>93978
Я имел ввиду что нибудь типа бетона
453 493980
>>93977

>Что такое ассимитричное ускорение?


Урони яблоко на пол. Вот тебе пример переменного ускорения.
454 493981
>>93979
Человеку в космосе делать нечего. Нет задач. следовательно и обитаемые модули не нужны.
455 493982
>>93977
Если не ошибаюсь, они возникают только при меняющемся ускорении. Вот у Земли, движущейся по орбите, ускорение направлено в сторону Солнца. Модуль примерно постоянен, а вот направление в течение года меняется. Но от Земли волны с гулькин хуй. Тут нужна либо гигантская масса (даже если ускорение меняется медленно), например две СМЧД, движущиеся по орбите в нескольких световых годах друг от друга. Либо охуенное изменение ускорения, скажем две нейтронные звезды (масса все же не крошечная), на расстоянии как Меркурий от Солнца друг от друга.
456 493984
>>93981
Для обывателей да. Раньше и когда Зеслю исследовали. Обвааиели оставались дома, и говрили, делать там нечего. А любопытные садились на корабли и открывали новые земли

>>93980
>>93982
Есть ли гипотическая возможность создания портативного генератора гравитации?
457 493985
>>93982

>Если не ошибаюсь, они возникают только при меняющемся ускорении.



Яблоко падает на пол. Когда яблоко летит то какое ускорение у него? А когда яблоко уже упало и лежит?
458 493986
>>93984

>Для обывателей да


Для ученых тоже. Роботы выполняют всю работу в космосе, в том числе и научную.
459 493987
>>93977
Не «либо», а «и» вообще-то. Массивное тело с переменным несимметричным ускорением.

Симметричные ускорения — например, сжатие/расширение сферы или её равномерное вращение — гравитационных волн не произведут.
460 493988
>>93984

>портативного генератора гравитации?


Не возможно.
461 493989
>>93987

>Массивное тело


Любое тело имеет массу вообще-то.
462 493991
>>93985

>Когда яблоко летит то какое ускорение у него?


g в сторону Земли, если пренебречь воздухом.

>А когда яблоко уже упало и лежит?


0 (если пренебречь Солнцем и всякой Луной).
463 493993
>>93987
Т. Е. Если бы земля не двигалась, а была на месте ровно, то и гравитации не было бы?
464 493994
>>93993

>а была на месте ровно


А что за место такое?
465 493995
>>93991
Короче, в момент падения яблоко генерирует гравитационную волну.
466 493997
>>93991
Следовательно яблоко в момент падения излучает гравитационные волны.
467 493999
>>93994
Точки лагранжа

Ну или гипотически. Если тело неподвижно, то и гравитации не будет?
468 494001
>>93999
Не путай гравитационные волны и саму гравитацию.
469 494007
>>94001
А какая между ними разница?
470 494009
>>94001
А если разонять небольшое тело ассимитрично так, по кругу, чтобы его суммарная энергия была как у Земли 10^29 дж, будет ли гравитация?
471 494016
>>94007
Разница как между статическим электрическим полем и электромагнитными волнами. Гравитация -- это искривление пространства (которое мы воспринимаем как притяжение), гравволны -- это распространение такого искривления.

>>94009
E=mc^2, так что будет. Даже фотоны гравитируют, из уравнений поля Эйнштейна это вытекает, там "масса покоя" не нужна, достаточно энергии.
472 494019
>>94016

>E=mc^2, так что будет. Даже фотоны гравитируют, из уравнений поля Эйнштейна это вытекает, там "масса покоя" не нужна, достаточно энергии.


Падажжи, но тогда получается, что в одной системе отсчета тело сворачивается в ЧД, а в другой нет, ведь энергия относительна. Как так?
473 494020
>>94019
А вот так.
Есть такая быстро движущаяся система отсчета, относительно которой твой анус - черная дыра. Но эта система отсчета сама в свою очередь выглядит для твоего ануса как улетающая со световой скоростью черная дыра.
474 494024
>>94016
Стоять.
Это что получается, я только шо гравитацию искусственную придумал?
475 494025
>>94020
Возьмем нейтронную звезду. Ей до коллапса немного осталось. Один корабль пусть мотыляет на орбите. Его скорость сравнительно невысока, для него нейтронка остается нейтронкой. Другой корабль пусть охуенно быстро (хз, 99,999% скорости света) улетает. Для него звезда приобретает дополнительную энергию и сворачивается в ЧД. Тогда он резко давит на тормоза, разворачивается и прилетает обратно. Как капитанам двух кораблей придти к согласию, осталась нейтронная звезда нейтронной звездой или превратилась в ЧД?
476 494028
>>94025

>Тогда он резко давит на тормоза


В этот момент ЧД для него разворачивается обратно.
477 494042
>>94028
За счет какого процесса? Насколько я знаю, ЧД могут худеть только излучением Хокинга. Но так или иначе, каким образом сингулярность (ну или что там под горизонтом событий) разворачивается не просто в буйство элементарных частиц, а в нейтронную звезду, которая обладает определенной структурой? (Или, если изменим начальные условия, в белый карлик, в обычную звезду?)
478 494050
>>93831
Пятна - показатель активности солнца. Меньше пятен -> меньше активности ->меньше излучения -> ебун.
479 494052
>>94024
Гравиволновый передатчик.
480 494059
>>94052
А в яем отличие от гравитации?

Слушай, а грави волна имеет мощность? Вот на каком расстоянии Земля прятигивает другте обьекты, скажем человека?
481 494070
>>94042

>За счет какого процесса?


За счет никакого. Просто до торможения это выглядело как черная дыра (застывший коллапсар) а после торможения снова засветилось.
Ты же не спрашиваешь за счет какого процесса происходит лоренцево сокращение?
482 494073
>>94070
Но ведь если для нас это ЧД, значит она ничего не может излучать. А капитан корабля на орбите мамой клянется, что эта хуйня (пусть это все же горячий белый карлик был) ни на минуту не переставала светить во все стороны. И раз скорость света постоянна, мы не могли от него "улететь", какие-то сильно смещенные в радиоволны кванты мы должны регистрировать.
483 494091
>>91856

>Космизм придумали в России, а не в США.


Ващет в Древней Греции.
484 494102
>>91209

>какая частица сейчас считается самой маленькой и неделимой - нейтрино или кварки?



Самая маленькая (маломассивная) частица - нейтрино. Ее масса - всего 0,28 эВ.
Sage 485 494118
>>94059
Пошел нахуй, тупой школьник
486 494119
>>94073
Вангую, что эти "сильно смещенные кванты" будут эквивалентны сильно смещенным квантам от пытающихся забежать за горизонт событий частей звезды. Ты к какой-то хуйне приебался, честное слово. Почитай вон про запутанность, там и не такое бывает.
487 494122
>>94102

>Ее масса - всего 0,28 эВ



Нобелевку получил уже за прямое измерение нейтринных масс? 0,28 эВ это просто ограничение сверху на сумму масс всех трех сортов (электронного, мюонного и тау), да еще и полученное очень косвенным путем.
488 494125
>>94118
А ты кто?
Покажи свою научную степень давай

Чмо диванное
489 494126
>>94059
Блядь, тебе написали уже, >>94001
>>94016
Ты понимаешь, чем волны в океане отличаются от самого океана?
490 494129
Правильно ли я понимаю что масса атома равна сумме всех составных частиц + энергия связи?
491 494131
>>94129
Именно так, причем более 99% приходится именно на энергию связи.

Не составных, а составляющих, неграмотный ты наш.
492 494146
>>94129
Смотри E=mc2. Понятно?
493 494149
Как в черной дыре может существовать объект с бесонечной плотностью, если масса чд вполне себе конечная ?
494 494162
>>94126
Волны это вода и океан тоже

В чем отличие грави волны от грави поля?
Множество волн порождает поле

мимо другой анон
495 494163
>>94146
Энштейн же жид пиздабол.
Спеыиалтно скрыл и вывел бредовую теорию, чтобы рокфеллерв смогли продавать нефть. Иначе кто бы ее покупал при халявной энергии?
496 494164
>>94149

>существовать объект с бесонечной плотностью


Прямо таки существовать? Прямо таки взяли и выяснили уже, что там за горизонтом? Ебать тут шизиков-наркоманов повылезло.
tinfoilhatday-4-1024x1024.jpg175 Кб, 1024x1024
497 494165
>>94163
Двачую адеквата. Тут кстати тоже все в заговоре с Эйнштейном, он им деньги перечисляет, чтобы они простому анону мозг пудрили. Вон где не спроси про эти теории носительности - несут только бред, время у них замедляется, пространство на гармони играет, а еще эти гомики-фотоны сцепленные, пиздец короче. Но ничего, скоро настоящая наука восторжествует, подожди братишка еще немного, скоро сам все увидишь.
498 494170
>>94149
Автоматически следует из нулевого размера при ненулевой массе.
499 494172
>>94165
Лол

Мировая элита тормозит прогресс, чтобы держать население в тупом виде, и чтобы оно потребляло и покупало. Но тупые дегенератв шутят про шапочку

Знаешь зачем форсят негров и еврору накачивают африкой? Чтобы плодились больше и потребляли. Чем глупее население тем выгоднее олигархам. Ваш космос им нахуй не упал
tinfoilhat-3.jpg149 Кб, 800x685
500 494174
>>94172

>Мировая элита тормозит прогресс,


А то бы я ебанул термояд в подвале, ты бы ебанул термояд в подвале и из воды в кране дейтерий бы сцеживали через марлю, вот зогжили бы.

>чтобы держать население в тупом виде


Для этого достаточно реформы образования. Вон я еще помню саентач, когда он над /b/ возвышался. А сейчас не всегда могу понять, там я, или тут.

>Знаешь зачем форсят негров и еврору накачивают африкой?


Да знаю, сам два года на негряном насосе работал по контракту, перекачивал.

>Чтобы плодились больше и потребляли.


А то без насоса они вообще из африки не хотят вылезать, пиздей они там ленивые.

>Ваш космос им нахуй не упал


Да, это заговор. Потому что иначе пришлось бы предположить, что при выигравшем капитализме тут сыграла роль экономика. Самому тошно от таких басней, двачую.
501 494176
>>94163
А причём тут Айнштайн? Лоренц и Пуанкаре тоже жиды?
502 494182
>>94162
Да, аналогии все только усложняют.
А электромагнитные волны - тоже вода?
503 494184
Наблюдается ли у каких-то объектов синее смещение или все только удаляются от нас ?
504 494186
>>94174
Ту глуп

В развитом обществе лбди хуже плодятся т. к. Ценности у них другие. Полэтому коренные европейцы вымирают. А то же время олихаргхам нужна быдла масса, которая будет потреблять и не задавать лишних вопросов.
Белый француз купит себе маленькую экономичную машину.
Араб купит себе йоба авто с 300 л.с. Чтобы было, и похуй что больше 130 км/ч по трассе нельзя. Кто из них выгоднее для автопроизводителя?

>>94176

Нет
Энштейн спиздил их идея и перевернул
14634611518520.jpg22 Кб, 480x270
505 494189
>>94186

>Ту глуп


Тулуп, неуч.

>В развитом обществе лбди хуже плодятся т. к. Ценности у них другие. Полэтому коренные европейцы вымирают.


Из чего следует логически, что надо насосом на свободное место накачать силком негров. Иначе никак.

>А то же время олихаргхам нужна быдла масса


Их не надо кнутом сгонять, они сами наползут, как тараканы. Что всегда поражает в зоговцах - это как объяснить психотронными лучами с нибиру то, что логически вытекает из текущих условий. Еще и отвергать сами условия, ниет, эта зогавар!11

>Кто из них выгоднее для автопроизводителя?


Тот, который взял кредит?
Ты такой толстый, что прям тупой.
Sage 506 494190
>>94184
Гугл отключили, хуесос?
507 494206
>>94189
А кто неграм платит за лодки для пересечения моря?

Лодки на пальмах не растут с моторами

Тем более берут негров массово, а не хохолов с русней
508 494207
>>94189

>кредит


А разница?
Прибыль от продаж идет

Кредит это выгодно для банков
509 494213
В сцай остался только рак, так что спрошу тут, не по теме
Существует ли технология, позволяющая создавать небольшие экраны, не более 1х1 см, произвольной формы? В форме там сердечка, крестика, хуя?
510 494262
>>94073
Каноническая ошибка. Сколько "энергии" не сообщай объекту, чья вторая космическая скорость меньше с, ЧД, объектом, чья вторая космическая больше с, он не станет. Собственно, это прямое следствие из Лоренца - как бы быстро ты не летел, если свет может покинуть тело, он будет лететь со скоростью с. А ты - чуточку, но медленнее.
511 494263
>>94213
Татьяныч такое сто лет как сделал. Произвольная форма экрана - за доп бабки. Если интерес предметный, оставь фейкомыло, я тебе продам такие дисплеи.
512 494265
>>94262
Ну то есть гравитирует масса покоя, а не энергия, зависящая от СО, так?
513 494266
>>94149
Я себе представляю (ибо никаких научных предпосылок нет) это так: под горизонтом сила гравитации (точнее, метрика искривленного П-В) такая, что пересиливает все остальные фундаментальные взаимодействия. Да так, что фактически никакие силы не могут оттолкнуть частицы друг от друга. В итоге, вся материя под горизонтом событий находится в одной точке пространства. Кстати, у Альбертыча одно из решений к такому и приводит.
То есть, горизонт событий имеет вполнеконкретные размеры, а вот материя под ним вся собрана в одной точке - геометрическом центре.
514 494268
>>94265
Смотри. Эквивалентность массы и энергии требует закона сохранения энергии, что в силу теорем Нетер требует однородности времени. Но в рамках ОТО (мы же про гравицапцию, верно?) Время в общем случае неоднородно, так как входит в соответствующие тензоры Э-И наравне с координатами, и дивергенция этого тензорав общем случае ненулевая. Поэтому источником икажения П-В является масса. Однако, перейдя к локальному случаю, мы получим эквивалетность массы и энергии а-ля СТО.
Я бухой,извини за сумбур
515 494269
>>94268

>Я бухой


Подозреваю, что ты толстый. Не в том плане, что несешь хуйню, скорее всего все верно. Но вряд ли ты не осознаешь, что твои собеседники с вероятностью 99% нихуя не поймут.
516 494274
>>94269
Я именно от такой хуйни и бухаю. Вот вчера у меня появился новый подчиненный, который начал сходу кукарекать за кефир. Я заставил его выводить матаппарат кефира, он два часа пыжился, но не смог. А потом я его удивил тем, что та же ОТО на самом деле тоже недоопределена - вводи любые координаты как условия и будешь получать нужные решения.
Ну блядь, каждое слово в посте гуглится, и есть причинно-следственная связь.
А когда я ему показал, как в домашних условиях на коленке проверитьвыкладки ОТО, у него был экстаз.
517 494279
>>92960
желтая, немигающая, юг. около горизонта, путь с юз до ЮВ.
ЭТО ЖЕ ЛУННА

https://staratlas.com/ а вообще посмотри чтоб на этом сайте стояло твое правильное местное время+ географическое положение. и ты легко отличишь сатурн от очень яркого юпитера
рог1.jpg100 Кб, 1200x628
518 494280
>>92581

>>на альфа Центавре можно будет построить атомную станцию. Радиотелескопы


Склады, космодром - своеобразный перевалочный пункт и плацдарм для полётов на Марс, места для проживания космонавтов.
519 494291
>>94280

>Руководитель НАСА считает, что все причастные к МКС страны, включая Россию, должны вместе помочь американцам вернуться на Луну. Это бы стало «по-настоящему международной коллаборацией».

1242px-JanOort.jpg183 Кб, 1242x1024
520 494297
Перекат во внешнюю Солнечную Систему:

>>494296 (OP)
>>494296 (OP)
>>494296 (OP)
>>494296 (OP)
521 494345
>>94025

> Возьмем нейтронную звезду. Ей до коллапса немного осталось. Один корабль пусть мотыляет на орбите. Его скорость сравнительно невысока, для него нейтронка остается нейтронкой. Другой корабль пусть охуенно быстро (хз, 99,999% скорости света) улетает. Для него звезда приобретает дополнительную энергию и сворачивается в ЧД. Тогда он резко давит на тормоза, разворачивается и прилетает обратно.


А второй капитан уже умер. И падает на чд.
522 495033
>>93485
total recall (1990)
Тред утонул или удален.
Это копия, сохраненная 29 сентября 2019 года.

Скачать тред: только с превью, с превью и прикрепленными файлами.
Второй вариант может долго скачиваться. Файлы будут только в живых или недавно утонувших тредах. Подробнее

Если вам полезен архив М.Двача, пожертвуйте на оплату сервера.
« /spc/В начало тредаВеб-версияНастройки
/a//b//mu//s//vg/Все доски